Jump to content

Wikipedia:Reference desk/Science: Difference between revisions

From Wikipedia, the free encyclopedia
Content deleted Content added
Line 574: Line 574:
Say that there is a planet with a very strong gravitational pull. Would the gravity condense some liquids into solid form, meaning there is more ice on the surface? Or would it cause more heat due to friction? If the strength of the planet's gravitational pull was the only differing factor, would the resulting surface conditions be more likely to be hotter, colder, or about the same as an Earth-like planet? [[Special:Contributions/97.104.210.67|97.104.210.67]] ([[User talk:97.104.210.67|talk]]) 05:57, 14 March 2010 (UTC)
Say that there is a planet with a very strong gravitational pull. Would the gravity condense some liquids into solid form, meaning there is more ice on the surface? Or would it cause more heat due to friction? If the strength of the planet's gravitational pull was the only differing factor, would the resulting surface conditions be more likely to be hotter, colder, or about the same as an Earth-like planet? [[Special:Contributions/97.104.210.67|97.104.210.67]] ([[User talk:97.104.210.67|talk]]) 05:57, 14 March 2010 (UTC)
:Gravity won't condense anything, pressure does that. Gravity is a factor in determining pressure, but it isn't the only one. Venus' surface pressure is about 90 times that of Earth despite having approximately the same surface gravity. --[[User:Tango|Tango]] ([[User talk:Tango|talk]]) 06:06, 14 March 2010 (UTC)
:Gravity won't condense anything, pressure does that. Gravity is a factor in determining pressure, but it isn't the only one. Venus' surface pressure is about 90 times that of Earth despite having approximately the same surface gravity. --[[User:Tango|Tango]] ([[User talk:Tango|talk]]) 06:06, 14 March 2010 (UTC)
::Excuse me for my mistake, I should have remembered that, but it is 1 AM my time. I'm just curious to know which result is more likely to occur. [[Special:Contributions/97.104.210.67|97.104.210.67]] ([[User talk:97.104.210.67|talk]]) 06:09, 14 March 2010 (UTC)

Revision as of 06:09, 14 March 2010

Welcome to the science section
of the Wikipedia reference desk.
Select a section:
Want a faster answer?

Main page: Help searching Wikipedia

   

How can I get my question answered?

  • Select the section of the desk that best fits the general topic of your question (see the navigation column to the right).
  • Post your question to only one section, providing a short header that gives the topic of your question.
  • Type '~~~~' (that is, four tilde characters) at the end – this signs and dates your contribution so we know who wrote what and when.
  • Don't post personal contact information – it will be removed. Any answers will be provided here.
  • Please be as specific as possible, and include all relevant context – the usefulness of answers may depend on the context.
  • Note:
    • We don't answer (and may remove) questions that require medical diagnosis or legal advice.
    • We don't answer requests for opinions, predictions or debate.
    • We don't do your homework for you, though we'll help you past the stuck point.
    • We don't conduct original research or provide a free source of ideas, but we'll help you find information you need.



How do I answer a question?

Main page: Wikipedia:Reference desk/Guidelines

  • The best answers address the question directly, and back up facts with wikilinks and links to sources. Do not edit others' comments and do not give any medical or legal advice.
See also:


March 10

crazy raccoon

http://www.youtube.com/watch?v=El_m4iitN5k&feature=related

i saw this with a bird too when i was a kid is that why they go in the middle of the street so they can get killed because there suffering to much and want to die i know its weard how i have said it —Preceding unsigned comment added by Thekiller35789 (talkcontribs) 00:04, 10 March 2010 (UTC)[reply]

It is not very likely that the animal in the video is attempting suicide-by-car. The consensus in the youtube discussion is that it was rabid, the symptoms of which include malaise, headache and fever, progressing to acute pain, violent movements, uncontrolled excitement, depression, and hydrophobia. I hazard they are more cogent reasons for the sort of behaviour seen on the video. --Tagishsimon (talk) 00:14, 10 March 2010 (UTC)[reply]

yes it could be rapid and mabye thats why its suicidal? like iv said iv seen this before in nature, with the bird i moved it out of the road and it just walked back into the road and waiting patiently til it got run over. which it did. —Preceding unsigned comment added by Thekiller35789 (talkcontribs) 01:39, 10 March 2010 (UTC)[reply]

See anthropomorphism. You are projecting human traits onto an animal. There is zero evidence that animals commit suicide in this way. There are any number of plausible reasons why an animal would be stupid enough to hang around in the middle of the road. I have to dodge stupid pheasants and grouse pretty much every time I drive my car, as they saunter into the middle of the road. I find it most easy to believe that they just have not got an understanding of the threat represented by my car. Quite why you reach for a suicide motivation is beyond me. --Tagishsimon (talk) 01:54, 10 March 2010 (UTC)[reply]
Is there a question here? Dismas|(talk) 05:15, 10 March 2010 (UTC)[reply]
Arguably yes: "is that why they go in the middle of the street so they can get killed". More succinctly, do animals commit suicide? At least, that's the one I've been answering. --Tagishsimon (talk) 05:21, 10 March 2010 (UTC)[reply]
And oops, sorry, that's with the exception of Bubbles the chimpanzee owned by Michael Jackson, who was alleged, according to our article, to have tried to commit suicide. And now we're on to something: Bubbles (chimpanzee)#cite_note-40 provides a bunch of references for a discussion of animal suicide. Who knew? --Tagishsimon (talk) 05:27, 10 March 2010 (UTC)[reply]

did you people read what i wrote im not talking about dodging pheasants or squirrels im talking about moving it out of the road and it just walked back into the road and waiting patiently til it got run over. which it did. —Preceding unsigned comment added by Thekiller35789 (talkcontribs) 05:32, 10 March 2010 (UTC)[reply]

That contradicts the description of the video, which says: "Just so you know...HE DID NOT GET HIT...A few people actually got him out of the road and back in the woods." --99.237.234.104 (talk) 06:47, 10 March 2010 (UTC)[reply]
He's now talking about the bird he mentioned later, which did indeed get run over. Vimescarrot (talk) 11:28, 10 March 2010 (UTC)[reply]
I'm talking about the general principle. Animals have little appreciation of the risks of the road. Find one on the road, shoo it off the road, and like as not it will wander back on the road. Maybe settle down in the middle of the road for a nice long sleep if it's warm. None of this is a suicide intent. I don't know how many times and in how many different ways I can say, just because an animal did something which if human would be taken as a conscious and premeditated suicide bid does not mean that it is the same for the animal. But whatever. If you want to live in some sort of manevolent Beatrix Potter world, be my guest; I'm done with this. --Tagishsimon (talk) 11:54, 10 March 2010 (UTC)[reply]
Also note that few, if any, animals are likely to fully understand death (that is, that they will cease to exist), therefore "suicide" isn't likely to have the same meaning for them. There may be animals that have instincts which cause them to do things which result in their death (like whales that beach themselves), but that doesn't necessarily mean they know that doing so will result in their death. (Although whales may be one of the few species that really do understand death, as we do.) StuRat (talk) 14:13, 10 March 2010 (UTC)[reply]
I have seen dogs and cats which appeared to be cognizant of the danger from cars. They cross the street in the middle of the block so they can be sure a car is not about to turn the corner and hit them. They time their crossing so avoid cars. Animals get old and sick and in pain, or perhaps they just get depressed. "Suicide by car" or "suicide by hunter" is not something I would rule out. It would be quicker and arguable less unpleasant than "suicide by not eating or drinking" which an elderly cat with cancer once did. Edison (talk) 17:44, 10 March 2010 (UTC)[reply]
They know that getting hit by a car would hurt, but understanding the possibility or meaning of their own death is probably beyond a dog or cat. As for not eating when sick, it's likely just a loss of appetite. StuRat (talk) 18:04, 10 March 2010 (UTC)[reply]
I'm with StuRat, much more likely the loss of abilities due to illness or old age would acount for death on the road. any number of illnesses of older animals could lead to loss of appetite. Richard Avery (talk) 19:33, 10 March 2010 (UTC)[reply]
The "few species that really do understand death" can't include humans. Just look at all the religious stories of the afterlife and you'll see that humans, as a whole, don't understand what death is about. Until relatively recently (a few thousand years ago), people didn't even know that consciousness rests entirely in the brain and that damaging the brain would end all thought processes. An animal would have no chance of knowing this. --99.237.234.104 (talk) 19:17, 10 March 2010 (UTC)[reply]
Well, perhaps some are intelligent enough to know that something more than just pain will occur, and that they will cease to exist (at least in the way they have existed). This requires a leap from seeing other dead animals and thinking that the same thing could happen to them. StuRat (talk) 19:45, 10 March 2010 (UTC)[reply]
Dolphins are commonly described as one of the few non-human animals that appears to actively commit suicide. Of course it is not (yet?) possible for us to know what they are really thinking, but they can at various times choose to do things that in other contexts they appear to know would kill them. --Mr.98 (talk) 18:52, 10 March 2010 (UTC)[reply]
I read in Encyclopedia Brown that iguanas are capable of ending their own life. ~AH1(TCU) 01:40, 14 March 2010 (UTC)[reply]

slouching

When I'm at the office I slouch and sometimes walk in a slouched position and I feel that it is a pain to adjust into a straight position. However, when I have to do moderate exercise (jogging, sprinting, abdominal exercises etc.) I noticed that after a while I'm involuntarily standing straight and its hard to slouch. Why is so? --121.54.2.188 (talk) 01:53, 10 March 2010 (UTC)[reply]

Slouching is a result of very relaxed muscles. Exercise tends to result in the opposite of relaxed muscles. --Tagishsimon (talk) 01:56, 10 March 2010 (UTC)[reply]
Could be you find your job boring. One is more alert and clear thinking when properly erect. The Alexander techniqueis about practising good posture and movement until it feels natural again.--Aspro (talk) 09:28, 10 March 2010 (UTC)[reply]
It could also be due to sitting in a bad position due to poor workplace ergonomics. Here is some more information:[1][2] --Normansmithy (talk) 15:52, 11 March 2010 (UTC)[reply]
Also see slouching. ~AH1(TCU) 01:35, 14 March 2010 (UTC)[reply]

Minkowski Diagram

In Newtonian physics for both observers the event at A is assigned to the same point in time.

This image comes from Wikipedia's article on Minkowski diagrams, and shows what such a diagram would look like in Newtonian physics. In the accompanying text, it says that the time coordinate in the ct and ct' are the same (as one would expect), but from the text it would appear that ct' is a little greater than ct. Help? 173.179.59.66 (talk) 04:54, 10 March 2010 (UTC)[reply]

I took the liberty to fix your red link. I'm not sure I understand your question. Dauto (talk) 05:12, 10 March 2010 (UTC)[reply]
OK, I think I understand it now. t and t' are the same coordinate, that is t=t' but they use different scales in the diagram. I other words, a unit of time t and is represented by a shorter segment than a unit of time t'. Dauto (talk) 05:42, 10 March 2010 (UTC)[reply]
Okay great, thanks. PS the article should be clearer. 173.179.59.66 (talk) 13:43, 10 March 2010 (UTC)[reply]

chrome sink

i unscrewed my handles from my bathroom sink today for cleaning and theirs a long brown metal rod attaching them. what is that metal its a dull dark brown? i at first thought it was brass but its darker and not shiny almost like lead. what is it?

Chrome plating is porous to moisture (and too expensive to be fashioned out of solid sheet) so not very good for sinks unlike stainless steel. Are you sure its chrome? They could be just cheap Ferritic Stainless Steel which has oxidised -after all their out of sight, so you just after durability. [3]--Aspro (talk) 09:33, 10 March 2010 (UTC)[reply]
Almost certainly brass that has just oxidised to be darker than "fresh" brass. --Phil Holmes (talk) 15:08, 10 March 2010 (UTC)[reply]


is brass what they plate chrome? —Preceding unsigned comment added by 67.246.254.35 (talk) 20:25, 10 March 2010 (UTC)[reply]

hello? —Preceding unsigned comment added by 67.246.254.35 (talk) 06:12, 11 March 2010 (UTC)[reply]

Chrome is what they use to make chrome. Brass (and bronze and copper) is another metal commonly used in sinks - especially for things like water connections. The last three oxidize to give green to greenish black to black. However if you have a bit of emery cloth (sandpaper), you can try rubbing off some of the oxidation. If it was brass/bronze/copper, you'll see the copper-colored unoxidized metal below the layer of oxidation. By the way, it may take us a while to get to your question - lag times of several hours are not unknown - we're all volunteers here. -- 174.21.235.250 (talk) 16:05, 11 March 2010 (UTC)[reply]

Inorganic Reaction Mechanisms

How could one tell whether a substitution reaction is proceeding via an associative rather than a dissociative reaction pathway? Is there a standard test? 188.221.55.165 (talk) 10:37, 10 March 2010 (UTC)[reply]

Yep. Just look at the entropy of activation.

From Dissociative substitution: "The entropy of activation is characteristically positive for these reactions, which indicates that the disorder of the reacting system increases in the rate determining step."
From Associative substitution: "The first step is typically rate determining. Thus, the entropy of activation is negative, which indicates an increase in order in the system."

Variations are also possible, notably the Sn1CB mechanism.
See User:Benjah-bmm27/degree/2/transition metal mechanisms and Eyring equation for more details.
Ben (talk) 13:48, 10 March 2010 (UTC)[reply]

weather

hOW —Preceding unsigned comment added by 216.178.90.202 (talk) 12:12, 10 March 2010 (UTC)[reply]

What you mean to ask? -- Extra999 (talk) 18:00, 10 March 2010 (UTC)[reply]
How to turn of the caps lock key? Richard Avery (talk) 19:26, 10 March 2010 (UTC)[reply]
I suppose he was to write "How to stop an hurricane?", but the hurricane was so fast... --Aushulz (talk) 19:45, 10 March 2010 (UTC)[reply]
We may never NOAA what the real question is. 10draftsdeep (talk) 20:16, 10 March 2010 (UTC)[reply]
If you want to know how weather works, I suggest you start by reading weather. If you have any specific questions after reading that article, we would be happy to help. --Tango (talk) 23:18, 10 March 2010 (UTC)[reply]
Also take a look at atmospheric dynamics, climate and Coriolis force. ~AH1(TCU) 01:34, 14 March 2010 (UTC)[reply]

Bubbles

When a soft drink is under high pressure, as in an unopened can, bubbles don't form. When the pressure is released, bubbles emerge from liquid. Why does high pressure prevent the CO2 from forming? 173.179.59.66 (talk) 14:06, 10 March 2010 (UTC)[reply]

The Bubbles ocupy a volume V and require an energy WORK=PV in order to form. At high pressure that energy is just to high and the bubbles won't form. Dauto (talk) 14:13, 10 March 2010 (UTC)[reply]
(ec) The CO2 doesn't "form" - there isn't a chemical change going on. The CO2 is already there, it's just dissolved in the liquid. However, the amount of a substance that can remain dissolved in a liquid depends on pressure and temperature. When you reduce the pressure by opening the container, the liquid can't keep so much of the CO2 dissolved - so it comes out of the solution as bubbles. You can see the same effect with (for example) salt dissolved in water. If you get some really hot water and dissolve salt into it until no more will dissolve - then let it cool - you'll see a lot of salt crystals form as the cooler water can't keep so much salt in solution. In that case it's heat that does it - but pressure works the same way. SteveBaker (talk) 14:16, 10 March 2010 (UTC)[reply]
Yeah, I had meant CO2 bubbles from forming...but my question is why higher pressure prevents gas from nucleating. I guess that there's some sort of equilibrium between the gas entering and the gas leaving, but I would have expected that you would just get bubbles going in and out, with no net gas difference. —Preceding unsigned comment added by 173.179.59.66 (talk) 15:53, 10 March 2010 (UTC)[reply]
I believe the reason for what you describe not happening is that the smaller a bubble of gas in liquid is, the greater the relative surface tension trying to shrink it is. Consequently, a tiny bubble has a hard time forming but, once formed, an easier time growing. This means that, ignoring temperature for the moment, above a certain pressure a potential bubble forming from dissolved gas would have insufficient internal pressure to overcome both external pressure and surface tension, so it cannot get started. If however the pressure drops sufficiently, and aided by nucleation sites if present, the bubbles once started will tend to grow further. A related phenomenon is the knocking sometimes heard in not-quite-boiling water caused by bubbles trying to grow but, due to turbulence, being deprived of steady-enough heat supply and collapsing again. I'm sure someone will be along soon to improve on or correct this hasty lay approach with all of that good stuff like proper physics an' maths an' graphs an' reference links. 87.81.230.195 (talk) 17:37, 10 March 2010 (UTC)[reply]
Well yes, that's pretty much what happens, although the bubbles never get large enough to see. StuRat (talk) 17:27, 10 March 2010 (UTC)[reply]
Are you talking about the surface tension of the liquid or of the gas?173.179.59.66 (talk) 14:53, 11 March 2010 (UTC)[reply]
The reason you don't see bubbles forming in a closed container, but you do in an open container is due to LeChatelier's Principle. Basically, when the system is closed, there is an equilibrium established such that the CO2 concentration in the headspace over the liquid and in the liquid itself stop changing. On a molecular level, individual CO2 molecules are both entering and leaving the soda in a closed bottle, but in bulk, the CO2 is basically not leaving or entering, the entire system is in balance. Because CO2 is a gas, when you open the container, the CO2 concentration drops, which throws off the equilibrium. Via LeChatelier's Principle, when you remove a substance from a stable equilibrium, the system will work to replace it. So the CO2 begins coming out of the solution to replace the lost CO2 gas. However, as long as it is open, equilibrium is never established, so the gas keeps coming out in bulk until the soda goes "flat". --Jayron32 21:56, 10 March 2010 (UTC)[reply]
Yes I understand that, but I would have expected that there would be bubbles going both way, as gas goes in and out of the fluid (as I mentioned above). To make sure I understood 87.81.230.195's explanation, does the increased pressure make the pressure inside the liquid increase as well, preventing bubbles from forming? Or is it the incoming gas molecules knocking the nascent bubbles and preventing them from fully developing? 173.179.59.66 (talk) 00:02, 11 March 2010 (UTC)[reply]
At equilibrium, the bubbles don't aggregate. There's no impetus to, as the flow happens as single molecules of CO2 move in and out at a dynamic steady state. Imagine it this way. If you picture a school building when class is in session. Occasionally, there's someone walking down the halls; like a kid leaving class to go to the bathroom, or another kid going back to class from the bathroom. The kids will tend to walk down the halls one at a time; there are always kids in the halls, but only a few, because the kids are pretty much staying where they are, and those in transit aren't likely to clump together. This is like the situation with the bottle closed. There is some movement of CO2, but the rate at which it is happening is relatively gentle, and counterbalanced by CO2 moving in the other direction. When the bell rings at the end of the day, well, that's like opening the bottle of soda. The kids ALL pile into the halls, and now their movement is very different than the occasional kid going to or coming back from the restroom. Now the kids all cram together and rush, in big clumps, out the nearest door. That's the same thing with the soda. Once there isn't an equilibrium, the CO2 in the soda is rushing out of the soda to reestablish that equilibrium. In that rush, they tend to aggregate and form bubbles because of the increased rate of "getting out" of the solution. --Jayron32 02:35, 11 March 2010 (UTC)[reply]
at equilibrium under pressure, the rate of exchange of CO2 molecules is slow and even - hence no bubbles. when the pressure is released from the top of the can, however, the CO2 comes out of suspension in the liquid at rate faster than can be transpired across the surface of the liquid; consequently you get pockets of gas forming inside the body of the liquid (aka bubbles). You can think of it as the liquid increasing its available surface area for transpiration - the surface of each bubble becomes a release area for dissolved gasses. it's basically the same process that causes the bends in diving (nitrogen in the air tanks is absorbed in the bloodstream under the pressure of the ocean, and turns into bubbles in the blood if the diver comes up too quickly), and why champagne is served in tall, thin flutes (decreased surface area at the top increases the number and life of bubbles formed). --Ludwigs2 03:14, 11 March 2010 (UTC)[reply]
@Jayron32: If the carbonated liquid were to be put under pressure by Nitrogen gas, then, by LeChat's principle, there would be an outflux of carbon dioxide and an influx of nitrogen. But there wouldn't be any bubbles. I don't think LeChatelier's principle is sufficient to describe the formation of bubbles; it's a statistical phenomena, but the carbon dioxide molecules themselves don't have any individual desire to re-establish the equilibrium. So the question becomes why does the outflux of carbon dioxide increase once the pressure above decreases?
@Ludwigs2: Why does the CO2 come out of suspension so fast when the pressure decreases? 173.179.59.66 (talk) 14:44, 11 March 2010 (UTC)[reply]
In other words, why is the nucleation of bubbles pressure dependent? 173.179.59.66 (talk) 16:26, 11 March 2010 (UTC)[reply]
In the case of pressure, a high pressure cooker similarly does not explode but at a sufficient pressure the water vapor contained inside needs to escape through a vent. ~AH1(TCU) 01:31, 14 March 2010 (UTC)[reply]

Structure of the earth

In this chapter of Journey into the Interior of the Earth one of the protagonists argues that a liquid layer in the interior of the earth will experience tides. (He presents this as an argument against the existance of a liquid layer as the displcements due to tides would cause daily quakes, which are not observed)

I know this is fiction, but does the outer core or mantel really experience tides? If it does, how much would the displacement due to tides be?

Would the model described in the chapter have sounded plausible to geologists in 1864(when the book was written)?

Brief summary of model:

-The earth is hollow.

-The heat in the interior is caused by water reacting with alkali metals.

-Since the heat source is located inbetween the surface and the centre, temprature would go up as we go down but then reduce as we go deeper than the heat source.

Also this model of a hollow earth:Structure of the Earth#Historical development of alternative conceptions, based on the flawed caluclations of the masses of the earth and the moon, assumes that the density of earth material will be the same as that of moon material. Was there any historical reason for this asumption?

Diwakark86 (talk) 14:36, 10 March 2010 (UTC)[reply]
Yes, the whole Earth experiences a tidal deformation due to the Moon's gravity; this note in Scientific American puts the tidal range at 25cm. That article also points to two presentations given by the operators of two particle accelerators, really the only people whose structures are large enough, and sensitive enough, to care. -- Finlay McWalterTalk 16:52, 10 March 2010 (UTC)[reply]
At the time you're talking about, scientists seem to be settled that the Earth isn't hollow - the calculations done before and around that time mentioned in the Age of the Earth article all assume the Earth isn't hollow. The Hollow Earth article is somewhat disappointing, in that it doesn't give any idea as to what general credence these ideas were given - it would appear that, as soon as people started seriously thinking about the interior of the Earth (in a scientific fashion) they thought of it as not being hollow. By the time of Murchison it was clear that the Earth was stratified to at least a nontrivial depth, but I don't think it was until large scale seismic studies were carried out (e.g. by Mohorovičić) a few decades later that the gross structure of the Earth became, to a limited extent, observable. As to the source of heat, Kelvin's calculations trying to age the Earth assumed it didn't generate its own heat, but that its warmth was a fossil of its creation (and thus put the Earth as being rather young) - but at that time no-one was at all clear as to how old the Earth was, and it wasn't until later that the mounting evidence of the great age of the Earth made it necessary that it contain source generating heat. I don't think anyone by that time would believe the third element of the model you discuss, as by that time physicists had a sufficient grasp of thermodynamics. So, all in all, I doubt that any scientist who seriously studied the known results about the Earth's interior would believe the model you discuss, but geology was a very young science with very few real practitioners, and with very limited means of finding things out. -- Finlay McWalterTalk 16:52, 10 March 2010 (UTC)[reply]
As for the Moon having the same density as the Earth, that depends on the model you use to explain their formation. If you believe they formed together from a dust cloud, then they should have a similar composition, although the heavier materials might have found their way down to the Earth and the lighter materials formed the Moon, so the Moon might be expected to be somewhat less dense (which it is). If you believe the Moon was captured from some other source, then it could be made from completely different material and thus have a completely different density. A more recent theory is that during the Earth's formation, or shortly after, a giant meteor hit that ejected the material which formed the Moon. In this case, the Moon might be a mix of the Earth's material and the meteor's, while the Earth would also have some of the material from the meteor, again leading to somewhat similar densities. You might think that the higher mass and hence gravitational attraction on Earth would lead to a greater density. It's actually the opposite, since most terrestrial material isn't very compressible in that range of g, and objects with less gravity tend to lose their lighter gasses and liquids to space. StuRat (talk) 17:23, 10 March 2010 (UTC)[reply]
As to the "Was there any historical reason for this asumption?" part of Diwakark86's question, there really wasn't much evidence to distinguish which of these theories of Lunar origin might be true (there's not nearly enough evidence now, alas). All they had to go on was the appearance of the Moon - it's vagely Earthlike, in that it has flat bits and mountainy bits, so one might as well assume, absent any better evidence, that its composition is vaguely Earthlike too. It wasn't until the examination of the Allan Hills meteorite in 1982 that science began to realise that actual physical samples of moonrock were already available on Earth (as Lunar meteorites). So, with what little evidence they had, it's a reasonable assumption for the geologists of 1864 to make. -- 87.114.240.222 (talk) 17:52, 10 March 2010 (UTC)[reply]
Even lunar meteorites, like the samples returned by astronauts, provide samples only from the outermost part of the Moon. If you only had samples from the surface of the Earth, they wouldn't tell you that the core is largely iron, or how big it is; likewise with the Moon's internal structure. --Anonymous, 21:52 UTC, March 10, 2010.
Also, plenty of earthquakes (some as deep as 500 km) occur in the mantle, as their depths indicate that they are below the crust yet they are still felt on land. ~AH1(TCU) 01:27, 14 March 2010 (UTC)[reply]

Biogas from plant and animal waste

Copied this from article space, where I am going to request deletion. I have let the page author know I have moved the question to here Gonzonoir (talk) 15:09, 10 March 2010 (UTC): I'm a boy from Botswana and i'm thinking of compiling a project on the comparisons of producing biogas between using rotten plant materials and cow-dung, so if you have got any piece of information which i can use be free to help me out and the god lord will bless you, i just want to make things better in my country and make my parents proud of my achievements. Originally posted by Ofentse mmereki (talk) 15:03, 10 March 2010 (UTC)[reply]

Optimum gas production per cubic metre requires a mix of both dung AND plant matter. The ratio is usually discovered by trial and error based on what is most available. See this section for more info:Anaerobic_digestion#Feedstock--Aspro (talk) 16:03, 10 March 2010 (UTC)[reply]
Back in the 1970's some third world country had a project to install a zipper in old truck inner tubes, to allow the insertion of dung. Methane would be produced in the tube and eventually it would be inflated to a pressure sufficient to allow the gas to be transmitted via a rubber tube to a gas ring on which the family could cook. The next day, open the zipper, clean it out, repeat the process. Surely someone has worked on this since the 1970's to improve it and make it larger scale. A larger scale version of the same plan might work like the gas producing plant in my great-grandfather's house from the late 19th century, but using dung and plant material to produce methane in place of the carbide he used to produce acetylene gas. A metal tank outside the house had a suspended or floating dome to maintain pressure containing the gas over a water reservoir into which the carbide was dropped whenever the pressure dropped below a setpoint. Metal pipe carried the gas into the house, wherein it went to gaslight fixtures and a cookstove in the kitchen. Please observe safety precautions with methane, since it is explosive, and the gas or its combustion products can cause asphyxiation. Commercial gas plants add an odorant so that leaks are detected, since methane has no odor of its own. I wonder if the dung and rotting plant material would provide their own odor when the gas was used? In a place where dung is commonly used for fuel, the odor should not be that novel. One important consideration is avoiding an explosive mixture of oxygen with the methane in a reservoir or in pipes. Some results at Google Book Search include (which discusses an improved methane generator), (which says the leftover slurry can be sold as fertilizer),(which says 5-6 bovines can provide 40 cubic meters of gas per day), (which has some simple designs), as well as [4], [5], [6], [7], and [8]. Edison (talk) 17:38, 10 March 2010 (UTC)[reply]
For Wikipedia articles, see biogas and methane#Uses. ~AH1(TCU) 01:24, 14 March 2010 (UTC)[reply]

Oh my god i never thought that things would get this complicated but i think it is for the betterment of my knowledge, can anyone please tell me how you can compile a project with two tanks, one containing cowdung and one containng rotten plant materials and then they must state the comparisons and diferences of the two like each's effectivesness, the amount of gas produced in each and how i can start compiling it.... i know it is too much but please i need serious help because it is a project. thnks.--168.167.134.115 (talk) 14:06, 15 March 2010 (UTC)[reply]

Heating through air resistance

I ride a motor-cycle to work, around 30 miles on freeways. Last few mornings it has been cold. This got me to thinking about heating through air resistance. Re-entering spacecraft experience extremely high temperatures, and even jet aircraft wing leading edges heat enough to make them a viable target for IR-based air-to-air missiles to lock on. So, my question is, how fast would I have to drive to heat up my hands to 37 degrees Celsius? Assume I'm at sea level and the airflow to my hands is completely unobstructed. --John (talk) 16:57, 10 March 2010 (UTC)[reply]

Aerodynamic heating is generally due to the air being compressed as an object passes through it. Per compressible flow, there is little compression (thus little heating) until Mach .3, about 230 MPH. Keep in mind also that moving air will speed up heat transfer away from your hands, making them feel colder. So I would expect your hands would be colder while moving for any speed that you're able to ride (solely considering aerodynamic effects). anonymous6494 17:13, 10 March 2010 (UTC)[reply]
When I pump up the tyres of my bicycle, the end of the pump gets hot. 78.151.126.97 (talk) 17:22, 10 March 2010 (UTC)[reply]
From memory the ‘frictional’ coefficient is 0.007 deg C per knot of wind speed. Small but is useful in meteorology.--Aspro (talk) 17:47, 10 March 2010 (UTC)[reply]
That seems too low. Based on that, and a knot (unit) being 1.151 mph, a plane going 1151 mph would only be heated 7°C. I suspect it's nonlinear, with more of an effect at higher speeds. StuRat (talk) 18:21, 10 March 2010 (UTC)[reply]
Right Stu, that coefficient is useful until the speed gets to be high enough for compressibility to start playing a greater roll. As anon said above, that speed is usually considered to start in the neighborhood of mach 0.3 or so. Googlemeister (talk) 19:14, 10 March 2010 (UTC)[reply]
Meteorology it is at sea level. (Motor cyckles not much higher). Otherwise it does go non linear (log ‘e’ I suppose). The batteries in my slide rule have gone flat, but there is a bit about this for those smarty pants who are good at mental arithmetic.[9]] Now lets see: carry one and add it to the first number you thought of... Foreseeable problems: High pressure turbulence will cause sevier bruising (Try skying behind a jet-ski in the wash of the water jet!) . Fighter pilots ejecting (or even bailing out) at high speed get bruised from air blast. The expanding air (after being compressed on the leading edges) will under go adiabatic cooling thus freezing the local tissue towards leeward side of the flesh. The local police forces will see an opportunity for issuing a months worth of speeding tickets on one night that this motorcyclist driveS though. Inquirer is obviously on a higher plane than the rest of us and would be better off in a Buddest monetary where he could write a squeal to the Zen and the Art of Motorcycle Maintenance.--Aspro (talk) 20:40, 10 March 2010 (UTC)[reply]
Also the NASA article points out that it is British Thermal Units (or Joules, Ergs, Watts etcs) that is the energy units to consider first and full most. The Temperature rise of air molecules in themselves does not contribute much to the question asked, which is about positive heat balance via increased linear speed.--Aspro (talk) 21:00, 10 March 2010 (UTC)[reply]
This is a tough one. Gut feel says that the answer is clearly going to be much faster than a motorbike could go - because the wind-chill effect of removing the 37 degC heat from your hand has to be overwhelmed. But we don't want 'gut feel' - so that won't do!
But we're not going to come up with a good number. Perhaps the best way to think about this would be from a thermodynamic angle. I guess we could figure out the coefficient of drag of the bike+rider, then we can calculate the drag force - and from that the amount of energy from the engine of the bike that's going into overcoming drag. That energy has to go somewhere - so we could consider what fraction might go into heating the air that the bike leaves behind in it's wake and what fraction is heating the skin of the bike and the rider - and make a stab at what fraction of that would be warming your hands. Knowing the energy input we should be able to figure a temperature rise. My big problem is that there could be three orders of magnitude of estimation in all of those guesses - so the answer might come out to be "Somewhere between 10mph and 1000mph." - which isn't really much use.
Another angle would be to consider things that actually do move very fast through the air. We know that at the speed of an airliner (400 to 600mph maybe), the skin heating is overwhelmed by the cooling effect of the cold air blasting past. Airliners might "ice up" but they don't get hot from the friction. On the other hand, aircraft that fly at Mach 3 and up have to be made of exotic metals and have active cooling on their leading edges to avoid melting. Similarly, spacecraft slowing down from orbit get incredibly hot when they are moving fast - but wind up being only slightly warm by the time they slow down and touch down. Meteors that people have found within minutes of hitting the ground as meteorites are reported to be merely warm, or even chilly to the touch. That strongly suggests that you're going to need to be moving considerably faster than Mach I...but again, so much depends on the air density and the shape of the object that we can't easily extrapolate from that to what you'd need on a bike.
Some questions can't be answered without an experiment - and this is probably one of them.
SteveBaker (talk) 00:53, 11 March 2010 (UTC)[reply]
Steve, to address your airliner point, airliners fly where the ambient temperature is between -40 and -70 deg, and air density is about 30% that of sea level, so there would be more then 3x as much heating if the aircraft was at sea level, but you are right, massive air resistance heating occurs at supersonic speeds, mach 0.9 has some heating, but it is a lot lower then mach 1.1. Googlemeister (talk) 14:26, 11 March 2010 (UTC)[reply]
Thanks a lot for all the great responses. So, we're looking at maybe somewhere between M1.1 and say M1.5? I know M2.02 is too high because I think that was Concorde's limiting factor to produce a TAT of 127 °C, which is way too hot for my fingers; and it flew at over FL50; as Googlemeister says, the heating effect would be greatly enhanced at sea level. I half-understand the calculations in the great NASA reference Aspro found, and I think we could solve it from that end with a bit of concerted effort. I also love the thermodynamic approach that SteveBaker brought in. Would it be fun to work it from the NASA data, and from the conservation of energy angle, and see if the two answers in any way fell into concordance? Or maybe folks have better things to do? I'd love to solve it, having wondered about it for several years. I predict the answer might be around M1.3. Let's take ambient air temp as 2 degrees C, a good deal warmer than at the heights aircraft fly at, as well as denser and damper. Any takers? --John (talk) 04:14, 12 March 2010 (UTC)[reply]
If the temperature outside your hands is cold, then wind chill would be a factor and would cool your hands more than they would warm them. The speed of a space shuttle during re-entry is much faster than the speed of sound, in which case your hands would not only heat but detatch from your arms. ~AH1(TCU) 01:20, 14 March 2010 (UTC)[reply]

Nanomaterials

what is the cause behind the invention of nanomaterials? —Preceding unsigned comment added by 118.102.186.3 (talk) 17:08, 10 March 2010 (UTC)[reply]

Bored scientists. Dauto (talk) 17:11, 10 March 2010 (UTC)[reply]
Bored scientists who had read There's Plenty of Room at the Bottom yet who (perhaps wisely) didn't pay too much attention to Engines of Creation.
Actually - it's not really "bored" scientists - it's scientists who are being paid to research things that can make the people who employ them very rich...which they are beginning to succeed at. SteveBaker (talk) 18:11, 10 March 2010 (UTC)[reply]
I suppose the reasons are similar to any other invention. That is: fame, money, to help mankind, etc. StuRat (talk) 18:14, 10 March 2010 (UTC)[reply]
What do you mean by "cause" here? Motivation of inventors? Or the scientific/technological path that led to them? --Mr.98 (talk) 22:39, 10 March 2010 (UTC)[reply]
See nanotechnology and carbon nanotubes. ~AH1(TCU) 01:18, 14 March 2010 (UTC)[reply]

(Arabic question)

أريد تقرير علمي عن تشريح الخشب النباتي —Preceding unsigned comment added by 95.170.208.114 (talk) 18:25, 10 March 2010 (UTC)[reply]

Hi. I don't know if you will be able to read my response, but all of our responses on here are generally in English. You could ask at the language desk but I still don't know if they will be able to help you with your question (which appears to be about reports on a wood plant, according to Google Translate). If you are unable to ask your question in English, then there are probably better places to ask it than here, as this is basically an English-only resource. --Mr.98 (talk) 18:47, 10 March 2010 (UTC)[reply]
مرحبا. أنا لا أعرف إذا كنت سوف يكون قادرا على قراءة ردي ، ولكن كل ردودنا على هنا عادة في اللغة الإنجليزية. هل يمكن أن نطلب من مكتب في اللغة ولكن ما زلت لا اعرف ما اذا كانت سوف تكون قادرة على مساعدتك في سؤالك (الذي يبدو أنه حول تقارير عن مصنع الخشب ، وفقا للمترجم جوجل). إذا كنت غير قادر على طرح سؤال في اللغة الإنجليزية ، ثم ربما يكون هناك أماكن أفضل ليطلب منه من هنا ، لأن هذا هو في الأساس باللغة الإنكليزية فقط على الموارد. -- السيد.98 (نقاش) 18:47 ، 10 مارس 2010 (تعم)

The above post translated by machine from Arabic:

hello. I do not know if I will be able to read my reply, but all our responses to here usually in the English language. Can we ask of the Office of the language, but I still do not know whether it will be able to help you in your question (which appeared to be about reports of wood factory, in accordance with a translator Google). If I was unable to a question was raised in the English language, and then there may be better places to ask him from here, because this is in the Foundation in English only on resources. -- السيد.98 (debate) 18:47, 10 march 2010 (prevail) Cuddlyable3 (talk) 10:24, 11 March 2010 (UTC)[reply]
Hmm. When you translate "talk" into Arabic and then back into English it becomes "debate". Think those translators might be a bit biased? Zain Ebrahim (talk) 11:59, 11 March 2010 (UTC)[reply]
Fun with online translators: Take any national anthem - translate to the language of your choice, translate it back again - enjoy songs like "The asterisks and strips indefinitely". 13:48, 11 March 2010 (UTC)
The lyrics were inspired by the patriot who said, "I only regret that I have but one asterisk for my country." -- Coneslayer (talk) 13:52, 11 March 2010 (UTC)[reply]
That second Arabic text appears to be a machine translation of Mr.98's post into Arabic, then back into English. The original question translates as "I want a scientific report of the anatomy of wood plant". Perhaps the question is about trees, in which case the OP could start at that article or use Google Scholar? ~AH1(TCU) 01:16, 14 March 2010 (UTC)[reply]

Prices of some raw materials

Hi all. I need to know the prices of these raw materials, that need for a case-study of a chemical plant:

  • Ethylene (99,9% mol)
  • Hydrogen chloride (99,8% mol)
  • Chlorine (99,9% mol)
  • Vynil chloride (99,9% mol)

I have another question: assuming that I'll buy more than 100000 ton/year of each one, I need to consider "spot price", "contract price" or what else? --Aushulz (talk) 19:02, 10 March 2010 (UTC)[reply]

Homework question?--Aspro (talk) 21:07, 10 March 2010 (UTC)[reply]
I am thinking that if you are buying 100000 tons per year, you should not be asking here! Issue a tender in the appropriate forum and see what the responses are. If you need a large quantity you may need a reliable supply and a contract. Check out Commodity market and spot price and futures market. Graeme Bartlett (talk) 09:01, 11 March 2010 (UTC)[reply]
Another thing that will alter the prices will be the required level of purity. 99.9999% pure materials cost much more the 99.9% pure ones. Googlemeister (talk) 15:21, 11 March 2010 (UTC)[reply]
@Googlemeister: You're right. I added requested purity.
Some other suggestion? --Aushulz (talk) 16:55, 11 March 2010 (UTC)[reply]

Dehydrated people

I seem to recall reading somewhere (don't think it was here) the percentage of dehydration needed before medics start giving intravenous fluids. Can someone help me find it please? --TammyMoet (talk) 19:10, 10 March 2010 (UTC)[reply]

Our article (Dehydration#Symptoms and prognosis) indicates that severe symptoms start to set in around the 10% mark. The Treatment section says that IV fluids would be given in emergency situations, or when severe symptoms are manifest. I'm not sure if there is a hard and fast rule as to when to administer IV (in contrast to orally administering fluids). Coreycubed (talk) 20:22, 10 March 2010 (UTC)[reply]
Thank you. There's such a lot of crap talked about hydration, I want to find some decent evidence, and that article might have some pointers. --TammyMoet (talk) 21:21, 10 March 2010 (UTC)[reply]

is it possible to express matter in the form of sound?

I read an interesting article that there are some specific "verses" which when uttered create specific sounds which represent the embodiment of desired matter. In simple words - hypothetically speaking - if i needed to see a person or procure a desired object by repeating some specific words/ sounds it is belived that the matter you seek or the person you seek materialises in front of you.

My question is can matter be represented as sound waves? —Preceding unsigned comment added by 213.130.123.12 (talk) 20:52, 10 March 2010 (UTC)[reply]

That doesn't sound like science, more like some weird science fiction. The closest thing to representing an object with sound waves would be the reflected sound from SONAR, where all of the waves, collectively, can be used to reconstruct the shape of the object from which they reflected. StuRat (talk) 21:05, 10 March 2010 (UTC)[reply]
A few things to bring us back to the realm of reality. No, matter is not sound. Matter is energy (see mass energy equivalence and wave particle duality for more details), however sound energy isn't of the correct "type" to properly contain matter information. Well, sort of. Via mass-energy equivalence, we can find the "mass" of a particular sound, if we know the energy that it represents (in sound, intensity is directly related to energy). But that doesn't mean we could represent actual matter as sound waves; that's because matter is energy plus. What matter is, is energy confined by quantum numbers. Quantum numbers are basically the information that tells one particle to be a proton, and another to be an electron, and another to be a neutron. If we convert that matter into energy, we can, from the energy, directly calculate how much mass contributed to the energy via E=mc^2. However, when matter turns into energy, quantum numbers are lost. So there is no way to recreate the form or shape of the matter from merely the energy information. You'd be missing the quantum number information. So, even if we could convert a chunk of energy into pure sound energy (and I suppose we could, there's nothing in the theory that would prevent it), there would be no way to extract from that sound energy the form of the matter in terms of how it was organized. The quantum number information is lost completely. So, you could tell roughly how much matter was converted to a sound, again via E=mc^2, but you'd never be able to reconstruct the form of that matter. There are "quasiparticles" of sound called phonons, but these shouldn't be considered particles on par with those of the standard model; instead they are merely the consequence of applying the mathematics of wave particle duality to sound waves. Treating sound as a particle works mathematically, but you cannot isolate or deal with a real "phonon" the way you can with, say, a "photon". --Jayron32 21:44, 10 March 2010 (UTC)[reply]
Jayron, everything you said is quite accurate except the part where you say a phonon is less real than a photon. Dauto (talk) 22:15, 10 March 2010 (UTC)[reply]
Sufficiently high energy gamma rays (photons) will decay into pions (a.k.a. matter) http://en.wikipedia.org/wiki/Greisen%E2%80%93Zatsepin%E2%80%93Kuzmin_limit . Maybe (?) a similiar mechanism exists for extremely high energy phonons? (after all, the sound wave is a consequence of electromagnetic interactions between neighbouring atoms in matter, and these interactions involve photons, which can decay into pions at sufficiently high energy ...) 83.134.158.50 (talk) 07:09, 11 March 2010 (UTC)[reply]
edited to add : certainly I'm not suggesting a human voice can generate phonons of sufficiently high energy to create pions, let alone to create a macroscopic object 83.134.158.50 (talk) 07:12, 11 March 2010 (UTC)[reply]

Were the words of these "verses" printed for you to try? No? I wonder why not! This is utter nonsense. If it were possible to create physical objects by uttering magical spells - I really think the major manufacturing companies of the world would be using large teams of carefully trained poets on all of their production lines - and they aren't. SteveBaker (talk) 13:44, 11 March 2010 (UTC)[reply]

It's possible to have pictures encoded within a radio signal. See slow-scan television. So in a way, yes, you can represent matter with sound waves. I think that's the closest you're going to get though. 75.157.57.12 (talk) 22:03, 11 March 2010 (UTC)[reply]
Ceci n'est pas une pipe... The image of matter is not matter. You could encode data onto a sound wave, and then decode the data at the other end, but you would get a picture not matter. And radio waves are not sound waves... they are light. --Jayron32 04:48, 12 March 2010 (UTC)[reply]

thanks for the particcipation folks and thanks to a few who called my question utter nonsense. firstly if man thinks he knows all then thats the first sign of his downfall, the greatest inventions of today were all called nonsense in the past. about these verses being printed or not yes ofcourse they are printed and they are tried by several people, they say the manifestation of mattter is abstract and it is for you to feel it and not see it. and as a parting shot, just because you cant demonstrate something, doesnt mean, it isnt true, maybe today we are unable to do this very soon we might be. A few years ago, if people had said we might be able to look and talk to people sitting in different planets by looking at our hand held phones, we would have laughed. a few years and now we realise where we stand. thats the beauty of science. thanks to the positive inputs from many people who see the crux of my question - as in the case of slow scan television we are now able to convert sound waves into picture - which is a big leap forward. if we can generate images from sound logically speaking or shall i say illogically speaking we should be technically able to actually generate matter from sound waves... there are texts in sanskrit in India which document this to have been done in the past - though if you ask me to demonstrate it i wudnt be able to. this is a scintific forum and we analyse areas and this is for sure in my humble opinion a very important hot spot for research. there are several things which existed in the past that had been lost forever because of so many reasons. if man thought modern science can explain everything and we start fooling ourselves into beleiving the scientific advances of the present day is the be all and end all of science, then we would be living in a fools paradise. if we look at our past we will find answers for the future.


March 11

Spinach and strength

It says on the article that spinach doesn't make you stronger because of a miscalculation in its iron content. But if it did have a lot of iron, would it make you stronger? I never heard of iron making a person stronger before.. ScienceApe (talk) 00:31, 11 March 2010 (UTC)[reply]

Please take a look at Human iron metabolism. Inadequate iron causes Iron deficiency anemia and will make one weaker. On the other hand, iron overload is toxic and won't make you stronger. 88.242.232.209 (talk) 01:11, 11 March 2010 (UTC)[reply]
You'll just get overwrought. Richard Avery (talk) 07:52, 11 March 2010 (UTC)[reply]
As opposed to cast down. Gwinva (talk) 08:01, 11 March 2010 (UTC)[reply]
Steel yourself against such ironic posts. Cuddlyable3 (talk) 10:58, 11 March 2010 (UTC)[reply]
Ferrous to answer this question, we need more information. SteveBaker (talk) 13:30, 11 March 2010 (UTC)[reply]
Ore you could experiment. Googlemeister (talk) 14:23, 11 March 2010 (UTC) [reply]
Popeye#Spinach says that "Early references to spinach in the Fleischer cartoons and subsequently in further stories of Popeye are attributed to the publication in 1870 of a study by Dr. E von Wolf which, because of a misprint, attributed to spinach ten times its actual iron content.[34] The error was discovered in 1937 but not widely publicized until T.J. Hamblin wrote about it in the British Medical Journal in 1981." - so the whole thing is down to a misprint. (That is so sad - the millions and millions of suffering children who were practically force-fed this disgusting so-called food by parents who were desperate to help them in later life...and it's all some simple misprint.) SteveBaker (talk) 13:40, 11 March 2010 (UTC)[reply]
I find spinach very tasty. So does my two year old. Dauto (talk) 14:42, 11 March 2010 (UTC)[reply]
Yeah, Steve, I think you're focusing too narrowly on a particular error here and missing the big picture, probably because of your personal dislike for the veggie. Spinach doesn't have as much iron as they thought, and anyway the notion that most people need more iron is a bit dated (iron is a necessary but rather toxic mineral and it's easy to get too much). But that doesn't mean you shouldn't eat it! Leafy greens have all sorts of things most people don't get enough of, starting but definitely not stopping with just plain fiber.
I also enjoy spinach, mainly raw. Try a salad made of spinach leaves, red bell pepper, a few Kalamata olives and a little feta. Possibly some avocado although that makes it less Greek. Pour some olive oil over the top, and then squeeze half a lime on it; you could sprinkle a little salt if you absolutely have to. Really really good! --Trovatore (talk) 20:41, 11 March 2010 (UTC)[reply]
Ah - one of those recipes. Basically, smother the noxious food item with as much strong smelling & tasting stuff as possible in order to avoid having to taste it! You can get your nutrition from all manner of other leafy greens - various cabbages and lettuces, brussel sprouts, bean shoots, leeks - you name it. You don't need spinach for health. SteveBaker (talk) 01:21, 12 March 2010 (UTC)[reply]
Most lettuce I find too mild. Iceberg in particular I don't want anything to do with. You can tell that its antioxidant content is low, just because it's not strongly colored. Antioxidants tend to be pigments (all those double bonds).
Cabbage, on the other hand, I just don't like. Brussels sprouts, broccoli, all those cruciform vegetables cruciferous vegetables, are palatable when cooked, but I don't care for them raw, and I think raw is probably best.
But my favorite leafy greens are the ones packaged as spring mix or herb salad — I like those even better than spinach. Still, spinach beats ordinary lettuce hands down. --Trovatore (talk) 02:02, 12 March 2010 (UTC)[reply]
Oh, and the recipe I gave is certainly not about drowning out the taste of the spinach — the spinach taste comes through, and I like it. If I didn't, I suppose iceberg would be OK. (Except I don't like the texture of iceberg, and since it has no taste, the texture is the only thing it does have.) --Trovatore (talk) 02:06, 12 March 2010 (UTC)[reply]
Steve, now I'm curious — do you taste phenylthiocarbamide? I was having a hard time imagining how anyone who objected to the taste of spinach, could even remotely tolerate cabbage. According to one of the articles I was clicking through yesterday, PTC tasters (like me; stuff's absolutely foul) are less likely to like cabbage. If you can't taste it, that could explain the difference in perception. --18:17, 12 March 2010 (UTC)
I'm not likely to go around tasting chemicals with long, dangerous-sounding names! I have no idea whether I can taste that stuff or not. Actually, my problem with spinach is that it really doesn't taste of very much at all...the texture bothers me some. But it's well-understood that taste is quite different between individuals. I know (for example) that most fish have almost zero flavor for me. People around me will be saying "Wow! This fish is so fresh!" - and I'm thinking "It tastes of almost nothing - just like almost every other fish I've ever tasted.". SteveBaker (talk) 01:49, 13 March 2010 (UTC)[reply]
Interesting -- it really doesn't taste of very much at all...the texture bothers me some is almost exactly what I'd say about lettuce (well, most lettuce). I like spinach precisely because it has more taste, and doesn't have that unpleasant pseudo-crunchiness. It's true though that spinach that's been left in the fridge too long acquires a very unpleasant slimy feel -- if that's what you associate with spinach, you just need to get it fresher. --Trovatore (talk) 02:46, 13 March 2010 (UTC)[reply]
I ate raw spinach and got Salmonellosis. 88.242.232.209 (talk) 23:04, 11 March 2010 (UTC)[reply]
Nothing in this life is free of risk. Yes, bacterial contamination is an ongoing issue. It would be a pity if that stopped people from eating veggies. --Trovatore (talk) 23:44, 11 March 2010 (UTC)[reply]
Raw spinach is great, that nasty stuff that comes in the can is fit only for a minor ingredient in lasagna. Googlemeister (talk) 14:09, 12 March 2010 (UTC)[reply]
I don't know that I've ever had canned spinach. I used to eat a fair amount of frozen spinach when I was a kid. My mom would boil it up and cut it liberally with mayonnaise. I really liked it, but I don't think you'd call that nutritionally recommendable. (Better than no veggies at all, but still better not to add that much fat and sodium.) --Trovatore (talk) 18:12, 12 March 2010 (UTC)[reply]
I think your mother knew what she was doing: doesn't the fat greatly increase the quantity of fat-soluble vitamins you get from the spinach, which are the chief benefit of cooked spinach to my mind? Better to eat your spinach with a little fat than with no fat. 86.178.167.166 (talk) 02:07, 13 March 2010 (UTC)[reply]
A little fat might help absorb carotenoids, according to our article on them. (Most people get enough vitamin K so that's not really a consideration.) But there's a difference between a little olive oil and a big old clump of mayonnaise. Just the same, it really was tasty. --Trovatore (talk) 02:14, 13 March 2010 (UTC)[reply]
For people that like spinach they might also like Amaranth or Callaloo as its known in the UK.
And please! Stop asking questions about food... it keeps me raiding the kitchen and putting on weight!--Aspro (talk) 18:47, 12 March 2010 (UTC)[reply]
If you're raiding the kitchen for raw spinach, I'm guessing that'll be OK. --Trovatore (talk) 20:34, 12 March 2010 (UTC)[reply]
Yeah - the projectile vomiting will solve the problem completely. SteveBaker (talk) 01:49, 13 March 2010 (UTC)[reply]
Re: the point above about broccoli being better raw, I would be careful before making such assumptions. I recall being told once about a study which found broccoli was best if lightly steamed, not raw. I can't seem to find that at the moment and if you do a research you may come across a widely reported study which I think is this [10] about raw broccoli being better then cooked (in particularly the bioavailibility of one compound being significantly lower). While a good study, it does use somewhat artificial example of 'raw' & 'cooked' "1 kg of raw broccoli was crushed with a blender and incubated at room temperature for 2 h prior to serving to the volunteers. One kilogram of broccoli was microwave cooked at 1000 W, crushed with a blender, and immediately serve." (Unfortunately it doesn't seem to say cooked for how long.) I'm not really sure why the crushing, it may given the small sample size (it's always going to be difficult to draw any definite conclusions from a sample size of 8) to try and prevent differences due to how well the participants chewed. Unfortunately that has the problem that there may be natural differences in how well people will chew the products if raw or cooked and incubating the broccoli for 2 hours after crushing is likely to cause differences compared to what most people consume when they eat raw uncrushed broccoli. Of course it may simply be crushing broccoli is common in the Netherlands.
In any case, this other study [11] found boiling was bad, but steaming, stir-frying and microwaving did not significantly reduce the content although this was of the vegetable instead of testing how it was absorbed like the earlier study did. This [12] mentions broccoli and other vegetables and goes briefly into the complexity.
Suffice to say as with most of these questions about which one is better health wise, it's unlikely to be a simple thing (although boiling the shit out of the things ala English and deep frying is unlikely to ever be good). In any case, even when raw is better, it doesn't automatically follow that it'll be better to just avoid the vegetable altogether, it may still be beneficial to eat it, along with other cooked and raw vegetables to ensure a variety of different nutrients.
(Personally I do enjoy raw brocolli on occasion, along with raw baby spinach, raw lettuce)
Nil Einne (talk) 15:57, 15 March 2010 (UTC)[reply]

Fluorescence in polar and nonpolar environments

It's said polar environments give emission with longer wavelength (i.e. red-shift) because "reorientation of polar solvent molecules compensates for charge separation", from my lecture notes. Why is that? My prof is so horrible and he has even worse notes. I looked at the wikipedia page for fluorescence but it doesn't explain these topics. —Preceding unsigned comment added by 142.58.129.94 (talk) 01:05, 11 March 2010 (UTC)[reply]

It sounds like you might be talking about Solvatochromism. Basically, a compound can absorb light, and in doing so, is changed to a higher energy form, that has different properties and structure than the lower energy form. Depending on the dipole moment of the lower and higher energy states, a polar or non-polar solvent may stabalize one or the other forms. This means the wavelength of light absorbed is dependent on the polarity of the solvent. Some compounds have a high degree of solvatochromism, such as Brooker's merocyanine.
However, I've never heard of fluorescence being related to solvatochromism, and the statement from the lecture notes means nothing to me. Perhaps a bit of context is needed: what are you studying right now? What class is it (i.e. upper level physical chemistry vs. freshmen biology)? What's in the solution (I assume you have a solution, as you're talking about polar and non-polar wavelengths environments)? Buddy431 (talk) 02:15, 11 March 2010 (UTC)[reply]

nitrile gloves

why do nitrile gloves smell like diarrhea? —Preceding unsigned comment added by 67.246.254.35 (talk) 01:34, 11 March 2010 (UTC)[reply]

Psychology. Namely, yours apparently. You believe them to smell like diarrhea, so they do to you. I think they smell something like nitrile gloves. --Jayron32 02:21, 11 March 2010 (UTC)[reply]


no iv asked others and they agree —Preceding unsigned comment added by 67.246.254.35 (talk) 03:24, 11 March 2010 (UTC)[reply]

I have to agree with Jayron32, they smell like nitrile. It's plastic-like mixed with a bit of sulfur. -- Flyguy649 talk 03:44, 11 March 2010 (UTC)[reply]

whatever the smell why do they smell that way shouldn't they be inert? its synthetic rubber —Preceding unsigned comment added by 67.246.254.35 (talk) 06:08, 11 March 2010 (UTC)[reply]

I'm not sure of the physiology behind smell, so don't know whether smell and reactivity should be related. However, rubber definitely isn't inert - e.g. it burns fairly readily (and smells awful when it does so!). 131.111.248.99 (talk) 11:42, 11 March 2010 (UTC)[reply]

yes i know rubber burns. u know what i mean by inert it shouldn't smell. obviously if u burn it it will. why does nitrile smell bad shouldent it just smell like latex or rubber? —Preceding unsigned comment added by 67.246.254.35 (talk) 23:22, 11 March 2010 (UTC)[reply]

advances needed to truly democratize the Internet?

I saw an article headline yesterday stating that most people consider the Internet (and by implication internet access) an inherent right. That led me to consider the recent events in western China where the government essentially pulled the plug on the Internet altogether. Similar events have happened in Iran lately. It seems, above and beyond any firewall/filter/proxy shenanigans, that if you go high enough up there is always someone who can just pull the plug and blackout a large swath of the Earth.

So, that led me to wonder about what sort of distributed hardware would be needed to prevent (or mitigate) this wholesale silencing? Knowing very little about the underlying structure of the Internet, the best idea I could come up with was interlinking WiFi hotspots that use solar/manually rechargeable battery sources. You would need an awful lot though to perpetuate a network on any useful scale... and there's certainly nothing to stop your favorite local authority figures from confiscating/banning such devices, so I readily admit it's not a very good idea. I know Wikipedia is not a crystal ball, but I hope I've given enough solid background to qualify this as a reasonable technology question. Succinctly put - how does the series of tubes become ownerless? 218.25.32.210 (talk) 01:37, 11 March 2010 (UTC)[reply]

By getting larger and being owned by many people. The problem isn't that they need to be "ownerless"—it's that you need them to have lots of owners, all of whom are interested in maintaining autonomy, none of whom want to be shut down by the government. (Or so a classical capitalist would argue.) If your entire economy is linked up to a lot of different service providers, that provides a rather resistant incentive against pulling the plug.
As for the technical suggestion... are you suggesting that the internet itself would be constituted solely on the WiFi linkages between individual machines? I don't think that would be robust enough unless you had an extremely high density of participating machines, and even in such a case, the density would have to be limited to local cities. It certainly wouldn't link up the world that way—China and Japan would never be linked by WiFi alone, since there is nobody sitting in the middle of the ocean with Wifi (or certainly not enough to have a worthwhile connection). Maybe at the edges you would have satellite connections or something, but you're going to be concentrating your infrastructure somewhere. (Concentrating it all in satellite connections is not necessarily a great idea if you afraid of governments, either—satellites can be shot down, can fail, can probably be hacked, etc.)
I'm not sure the answer to this is technological in nature, or at least not limited by the networking technology. Connecting countries requires big cables. A government presumably knows where these are. They could, in theory or in practice, shut them down or monitor their use. Increasing the number of cables can make this harder, but not impossible (it is still a finite number of cables). Increasing the number of participants would make shutting them down more problematic in some countries (but not all). A more robust solution might be to mingle political traffic with economic traffic in an invisible way (e.g. through encryption)—with a goal to make it so the government can't shut down political speech without a significant sector of their entire economy. (Note that I would not call this "democratizing" the internet. You are making it more robust, but that's not the same thing as "democracy.") -Mr.98 (talk) 03:26, 11 March 2010 (UTC)[reply]
Additionally, the novel Cryptonomicon proposes something along these lines, relying primarily on strong encryption as the means to ensure long-term communications/economic freedom. --Mr.98 (talk) 04:27, 11 March 2010 (UTC)[reply]
(e/c) This is actually not strictly a science question, but I'll answer it anyway. the internet was designed from its inception to be (technically speaking) a surveillance system. What I mean is that the client/server architecture is geared towards more-or-less static sites which are viewed by an assortment of viewers. Pragmatically, what this means is that anyone with the proper access can restrict and monitor everything that is viewed - the static servers have to advertise their locations on the web so that viewers can find them, so authorities always know where people are going to be looking. for an analogy, a campus I know was specifically designed with one building for all student groups and meetings, and that building was designed with (physical) firewalls that could be closed so that campus police could quickly and easily isolate and contain any student protests.
Peer-to-peer networking has started to break that client/server architecture up a bit - there were some people using peer-to-peer to get information and videos out during the Iran election troubles, and we all know the problems that the music industry has been trying to gain authoritative control over. but even with peer-to-peer a dedicated group with the proper access can track down, monitor, and suppress internet traffic. There's really nothing anonymous on the internet: best you can hope for is to throw enough sand over your electronic tracks to confuse anyone watching.
Internet democracy (should such a thing ever be tried) would require some legal guarantees to certain kinds of internet privacy and freedom. this has been specifically excluded in nations like China, Iran, and the US where the government explicitly retains the rights to monitor digital communications. I think the EU has some (weak) guarantees in place, but I suspect it will be decades before legal ethics catches up with changes in technology, so I wouldn't hold my breath. --Ludwigs2 03:40, 11 March 2010 (UTC)[reply]
How about dial-up ? That is, if the people in China can call outside the country and connect to an ISP there, they can stay online. Of course, there are long distance fees for that, but hopefully only pennies a minute. During their next Tiananmen Square massacre, they could connect up to get the real news and download their pics. Now, China could always block all international calls, but that would certainly hobble the economy. They might just block calls to phone numbers owned by ISPs, though. It would be nice if everybody outside China had a way that, when they get a call from China, it would just hook them up to the Internet automatically. I suppose China could also develop snooping software that would listen to every call, and disconnect anything that sounds like data transmission, but that would probably also knock out faxes, which are used in business. StuRat (talk) 06:14, 11 March 2010 (UTC)[reply]

So the problem is that if a government decides it doesn't want its people talking to the rest of the world - then nothing involving wires or optical fiber would solve the problem (if it is a problem...which we can discuss) because those wires can be cut/filtered/monitored at the borders. Radio signals (like Ham Radio's Packet radio) are lacking in bandwidth - and can still be monitored - and perhaps even jammed. What is needed is a wireless - narrow-beam technology that's hard to intercept, impossible to filter and impossible to jam. That would require some kind of "direct-to-satellite" approach where you'd send your data via narrow-beam radio signals or lasers aimed at a satellite and the results would be beamed back using normal transmission mechanisms. Such a system would be horrifically expensive - and would require someone with deep pockets to launch and maintain the satellites. The problem with that is how it gets paid for. Individuals couldn't pay for it using credit cards because those kinds of transactions are easily spotted in the local banking system. It would have to be advertising-funded, but that still leaves the system open to blocking by a government banning the sale of products and services that advertise over this subversive communications system.

The real solution is that people who don't want an oppressive government who shuts down their internet have to get rid of that government. In the end, that's the only way. If you have an oppressive government then all of the technological solutions in the world won't help because they can simply ban use of whatever device is required to access it - clamp down on the importation or manufacture of such gizmo's and use secret police to track down and imprison people found using them.

Bottom line: This isn't a technological problem - and you can't solve it with technology either.

SteveBaker (talk) 13:25, 11 March 2010 (UTC)[reply]

It isn't an issue of 'cutting wires'; the issue is that the internet relies on identifiable numeric addresses as distribution points. If you know what addresses are used for distribution, you can control them. wireless and satellite systems will not solve the problem unless they somehow bypass the IP infrastructure of the net, because otherwise a source will still have an identifiable internet location that can be blocked. --Ludwigs2 18:02, 11 March 2010 (UTC)[reply]

If you're happy with high latency, IP over Avian Carriers and sneaker net (which can use IP over Avian Carrier technology with a slightly different physical layer) offer very high bandwidth with modern data storage technology. Very small high capacity memory cards are incredibly easy to smuggle across borders, so very little packet loss (which is important when each packet is gigabytes in size). But the latancy sucks. --Polysylabic Pseudonym (talk) 04:34, 12 March 2010 (UTC)[reply]

Cooking oatmeal

Why does it seem to take longer to cook oatmeal when you add the sugar before microwaving than if you cook it, then add the sugar? —Preceding unsigned comment added by 66.112.225.118 (talk) 05:43, 11 March 2010 (UTC)[reply]

Sounds like another homework question. To brake the bonds so that the crystals dissolve, is in this case Endothermic. You can even freeze water with a mixture of photographers hypo!--Aspro (talk) 08:38, 11 March 2010 (UTC)[reply]

Note:, you’d still better read up on it. This is only half of what you need to know but was provided in case your completely stuck, but it is not the whole answer.
Welcome to the Wikipedia Reference Desk. Your question appears to be a homework question. I apologize if this is a misevaluation, but it is our policy here to not do people's homework for them, but to merely aid them in doing it themselves. Letting someone else do your homework does not help you learn how to solve such problems. Please attempt to solve the problem yourself first. If you need help with a specific part of your homework, feel free to tell us where you are stuck and ask for help. If you need help grasping the concept of a problem, by all means let us know. --Aspro (talk) 08:57, 11 March 2010 (UTC)[reply]
I'm sorry but this really doesn't sound like a homework question to me, more like a question arising from some piqued curiosity, from observing what happens to porridge in the microwave. I'd personally appreciate a good answer to this question - and my homework days are long gone! So would you please tell me, even if you won't tell the OP? --TammyMoet (talk) 10:06, 11 March 2010 (UTC)[reply]
It is important to understand this phenomena before mixing together substances that may cause one’s bench apparatus to undergo rapidly disassembly. So I will link to a pdf of a Dissolution Reactions Pre-Lab Script for your further edification - the answer is out there -enjoy! [13]--Aspro (talk) 11:20, 11 March 2010 (UTC)[reply]
(edit conflict) I agree with Tammy: this seems unlike a homework question. Here's my explanation, 66: when you heat something up, its temperature will rise to a different extent, given the same amount of energy, depending on the material. The energy needed to raise a certain amount of a substance by a certain number of degrees is called its specific heat capacity. Water takes about 4.2 joules to raise one gram by one degree Celsius, for example. When you cook the oatmeal alone, you only need to put in enough energy to heat up the oatmeal to the right temperature. You then add room-temperature sugar. But when you add the sugar into the oatmeal and cook the mixture, you've got to put in energy to heat up the sugar too - I'm not sure how its SHC compares to water or oatmeal, but still, it's going to mop up some of the energy that would previously have just heated the oatmeal. You need to put more heat energy into your food, to get it to the same temperature as your plain oatmeal, as a result. Since microwaves operate at constant power (energy transfer per second), you just leave it on for longer. In a nutshell: you have to heat up the sugar too. I could possibly have explained it without the SHC link but it would make interesting further reading for you. Brammers (talk) 11:54, 11 March 2010 (UTC)[reply]
Sugar is a poor absorber of microwave energy – instead it will take up heat from the water in the porridge. The specific heat capacity of the sugar will only take a few seconds worth of power (each spoonful = about 5 grams, or in other words a very, very small percentage of the whole total SHC). As the pdf suggests: the braking of bonds simply requires more energy than any energy released by the sugar molecules entering the solute and thus slows down cooking. However, the cooking instructions that I have seen on the side of the boxes always seem to be very generous as to cooking time required. Nor have I ever seen any suggestion to increase cooking time if sugar (or salt if you live in Scotland) is added first. Porridge isn’t so touch or go as a soufflé. The difference, I would imagine, is so small as to be unnoticeable unless set as a homework question. Maybe if one was cooking a very big goldilocks and the three bears size pot of it, it might be more noticeable but then microwaving that amount is a very inefficient use of power. I am asuming that this involves pre-steamed Quaker Oats type oatmeal, as ordinary rolled oatmeal takes so long to cook that the effect would be very difficult to measure. --Aspro (talk) 12:35, 11 March 2010 (UTC)[reply]
This is the original poster. This is not a homework question jeez. And it doesn't seem to be significantly colder after cooking the same amount of time, it just takes longer for the oats to expand and remains soupy longer. I was wondering why. 66.112.225.58 (talk) 05:18, 12 March 2010 (UTC)[reply]
In that case you may be witnessing what might best be understood by read Osmosis. In short: Sugar is hygroscopic. That means is will slow down the hydration of the oatmeal. By how much I don’t know, but if you prepare two bowls at exactly the same time, one sugared and one without, then leave both of them at room temperature. Both will hydrate whilst cold but you might find the sugared bowl ‘remains soupy longer’. So it might not be the cooking time that’s slowed but the hydration time that’s delayed. --Aspro (talk) 15:56, 12 March 2010 (UTC)[reply]
This may also be relevant to the fact that burning sugar does not work. ~AH1(TCU) 01:00, 14 March 2010 (UTC)[reply]

expansion of the universe

If exponential expansion of the universe can not be ascribed to a polar force of gravity then how about a force like centripetal force; i.e., expansion being due to the universe spinning? 71.100.11.118 (talk) 09:49, 11 March 2010 (UTC)[reply]

Spinning relative to what? How do you explain the expansive force at right angles to the plane of the alleged spin? Cuddlyable3 (talk) 11:02, 11 March 2010 (UTC)[reply]
I'm continually amazed that people come up with these 'classical' ways to try to explain such phenomena. Do you really think scientists wouldn't have already thought of trying such a simple explanation? Of course they did! But it had some horrible flaw and was discarded in favor of the much less intuitive answer that we have today. If there were a simple "classical" explanation, then Occam's razor would certainly have us using it instead of the theory we currently subscribe to.
In this case, there are a bazillion problems - there would be coriolis forces (which we do not observe) - there would be the question of where the center of rotation would be - because the expansion of the universe would look very different from the perspective of someone out on the edge of this rotating object than to someone who just happened to be in the middle. The idea that the earth would just magically happen to be in the precise center of the expansion is just vastly too improbable and smacks of geocentricism - which went out the door in the 16th century! Also, the universe is three-dimensional (at least) - if it were spinning about (say) a "North-South" axis like the earth does - then the expansion would only be in a plane parallel to the "Equator" - but here in the real universe, all objects are moving away from us - including the ones vertically above and below us which would experience no centrifugal force in a simple rotating universe. There are (I'm sure) dozens of other reasons why this explanation won't fly. But since all it takes is one flaw to destroy a hypothesis - this one is busted and needs no further investigation. SteveBaker (talk) 13:01, 11 March 2010 (UTC)[reply]
All it takes to fix one flaw is some handy dark matter! Also, I like your tautology "[geocentricism] smacks of geocentricism". Aaadddaaammm (talk) 13:53, 11 March 2010 (UTC)[reply]
There is a simple classical (i.e., non-quantum) model of the accelerating expansion, and it does involve a "repulsive force" that's proportional to distance like the original poster imagines. That's not the problem. It's not hard to write down a law that fits the data; the question is, why that law? There are two terms in the force law; what physical principle leads to those terms being present and other terms being absent, and what principle sets the value of the constant factors on each term (the force constants G and Λ)? Quantum field theory does seem to predict a lower bound for a certain unitless combinations of those constants, but the prediction is ludicrously higher than the observed value; no one can find a reason why the real-world value should be so small. That's the problem.
Centrifugal force doesn't work because it would lead to expansion only perpendicular to the rotation axis, which isn't what's observed; but the other problems Steve mentioned don't exist. Rotating cosmological models don't have a center of rotation, just as the expanding models don't have a center of expansion. The rotation axis has a direction but not a position. But we wouldn't be any better off anyway if the data supported a rotating cosmology, because we still wouldn't know why it's rotating at that particular rate along that particular axis. If there were evidence that the universe was rotating, physicists wouldn't be asking, "hey, what if the universe is rotating?"—they'd be asking "why is the universe rotating?". And that's what they're asking now: why is the expansion accelerating? And, incidentally, why isn't it rotating? -- BenRG (talk) 20:49, 11 March 2010 (UTC)[reply]

Well whenever there is an explosion in the universe like a supernova things slow down and come to rest. I assume this is in part due to the gravity of the remnants in the presence of empty space. The classical model of exhaustion of the energy from the explosion fits and this is what you would expect from a theory like the Big Band. This leaves a lot of questions that require lots of speculation just like a blind man trying to work his way through unknown territory ahead.. One idea I can't find considered is the possibility the things like permissivity might change in opposition to thinking according to classical physics. The idea applied here would mean that a greater distance can be traveled in the same amount of time the further from the Big Bang in time and space the distance is or that gravity has a polar force we can call anti-gravity. What are the flaws in these theories other than that they step on the toes of the classical or current model? 71.100.11.118 (talk) 15:01, 11 March 2010 (UTC)[reply]

Always ask, is there a way to falsify my theory? Until you've done that you've not a scientific theory. Imagine Reason (talk) 15:35, 11 March 2010 (UTC)[reply]
If the universe were spinning, that would assume that the universe is spherical or something similar, but the current theory is that the universe is flat and has infinite volume so there is nothing for it to spin relative to except for perhaps time as space is contained inside (but then again time might not exist outside the universe or there may not even be an outside). ~AH1(TCU) 00:30, 14 March 2010 (UTC)[reply]

nuclear candle

sir, can we model a nuclear fuel rod by using a common candle.many articles referring to CANDLE are found like. Potential of CANDLE reactor on sustainable development and strengthened proliferation resistance ETC

REGARDS SCI-hunter (talk) —Preceding undated comment added 11:07, 11 March 2010 (UTC).[reply]

CANDLE is in capital letters because it's an acronym, it stands for "Constant Axial shape of Neutron flux, nuclide densities and power shape During Life of Energy-producing reactor" - which has to be one of the most contrived acronyms I've ever seen! (But, I suppose, more memorable than a CASONFNDAPSDLOEPR reactor). Anyway, it has nothing whatever to do with wax cylinders with a bit of string up the middle. If you're still interested in "CANDLE reactors" then you can read more about them here. SteveBaker (talk) 12:45, 11 March 2010 (UTC)[reply]
Also known as a Traveling wave reactor --Aspro (talk) 12:51, 11 March 2010 (UTC)[reply]
From what I read - it seems to be a particular kind of TWR (ie not all TWR's are "CANDLE reactors" - but all CANDLE reactors are TWR's) - but the literature is very opaque if you're not an expert (and I'm certainly not!). SteveBaker (talk) 13:02, 11 March 2010 (UTC)[reply]
As both burn from one end to another ( like a candle) there is plenty of opportunity for confusion. I have put “CANDLE reactor” on the CANDLE disambiguation page but will hold off creating and redirecting it to the TWR article. --Aspro (talk) 13:10, 11 March 2010 (UTC)[reply]


okay i understand wax candles have no significance with a CANDLE in above reference i have made.But can i have any reference of modeling a fuel rod's nuclear reaction by a candle.Modeling neutrons in reactor by air around candle,the products of nuclear reaction by flame in a candle.Even if no proof is availaible,is that comparision apt.

Thanks in advance 218.248.11.214 (talk) —Preceding undated comment added 21:27, 11 March 2010 (UTC).[reply]

No, I don't think so. They are very different physical processes. --Mr.98 (talk) 22:57, 11 March 2010 (UTC)[reply]
I agree - there is really no kind of analogy at work here. SteveBaker (talk) 00:50, 12 March 2010 (UTC)[reply]

Shielding against Electro magnetic induction

How do you design a Faraday Cage to shield against interference from heavy C Band radio frequencies and what is the recommended material for the cage. —Preceding unsigned comment added by Shielding Buff (talkcontribs) 11:47, 11 March 2010 (UTC)[reply]

I believe a Faraday cage must be made of an electrically conductive material, so a wire mesh would be used. As for the ideal spacing of the wires, I will let others answer that. StuRat (talk) 13:33, 11 March 2010 (UTC)[reply]
The cage has to completely enclose what you are trying to shield. The rule of thumb is that the size of the holes is 10% or less of the wavelength. So for a 6 GHz C band wave, you would have a wavelength of 5 cm and a mess spacing of 5 mm. (Or do you mean citizens band ?) Thick copper plate would do the job, but steel plate would be cheaper. Security screen mesh or the fine grade of chicken wire should do. If you leave a hole, the em wave will travel over the surface and in through the hole via diffraction. So the tin foil hat is ineffective. Graeme Bartlett (talk) 18:33, 12 March 2010 (UTC)[reply]

Birds fall out of the sky?

What could cause the phenomenon described in this article? [14]

Aaadddaaammm (talk) 13:42, 11 March 2010 (UTC)[reply]

A large release of gas in a localized area, not enough oxygen for the birds to breathe. —Preceding unsigned comment added by Fire2010 (talkcontribs) 14:01, 11 March 2010 (UTC)[reply]
(edit conflict)Turbulent weather is typically the reason for raining animals. The following is an extract from the article:
Doppler Image from Texas showing the collision of a thunderstorm with a group of bats in flight. The color red indicates the animals flying into the storm.
In the case of birds, storms may overcome a flock in flight, especially in times of migration. The image to the right shows an example where a group of bats is overtaken by a thunderstorm.[1]. The image shows how the phenomenon could take place in some cases. In the image, the bats are in the red zone, which corresponds to winds moving away from the radar station, and enter into a mesocyclone associated with a tornado (in green). These events may occur easily with birds in flight. In contrast, it is harder to find a plausible explanation for rains of terrestrial animals; the enigma persists despite scientific studies.
Zain Ebrahim (talk) 14:03, 11 March 2010 (UTC)[reply]
Who knows. turbulence of the type described above is vanishingly rare in Somerset, UK. As the article says it is reported to have happened happened in several places in the world in the last year or two. There seems to be a strong clue in the second paragraph where it is reported that many of the birds had broken wings, beak, legs and internal damage. In my experience it requires more than gravity to cause that sort of damage in a bird, particularly a smallish bird like a starling. The report contains a likely scenario for the horrendous carnage incident and the force of the birds flying downwards in a panicked attempt to evade a predator could account for the birds' injuries. What is more interesting is the end of the report where the witness describes what she found. ... "like a scene from a horror film like Hitchock's 'The Birds'", well, no, because all the birds were alive in the film. Richard Avery (talk) 14:09, 11 March 2010 (UTC)[reply]
The OP's link refers to a "whoosing sound", which makes me think of strong downward winds, perhaps a microburst. StuRat (talk) 14:35, 11 March 2010 (UTC)[reply]
Hmm some interesting possibilities. The predator evasion theory doesn't sit too well with me - I give these little birds more credit than flying into the ground to avoid being eaten. I noticed that bit at the end too, Richard - it's kind of the opposite of "The Birds". Aaadddaaammm (talk) 14:53, 11 March 2010 (UTC)[reply]
A lot of birds did (fictionally) die in the film, from smashing themselves into buildings and such to get at the people inside. Deor (talk) 17:44, 11 March 2010 (UTC)[reply]
A bird falling out of the sky is one of the imageries used in the Dutch film De Vierde Man to symbolise the death of the black widow's 2nd husband. --Kvasir (talk) 18:04, 11 March 2010 (UTC)[reply]
The bird of prey theory sounds silly to me, too. StuRat (talk) 17:56, 11 March 2010 (UTC)[reply]
Birds falling out of the sky is nothing, try | fish falling out of the sky. --Kvasir (talk) 18:00, 11 March 2010 (UTC)[reply]
Fish falling out of the sky is easier, that pretty much has to be from a waterspout. StuRat (talk) 18:03, 11 March 2010 (UTC)[reply]
Although this sounds logical, and is the answer usually resorted to by meteorologists and others attempting to explain away anomalous falls, most of the many instances of such fish, etc falls seem to have occurred with no waterspout, or even the sort of weather in which waterspouts commonly occur, in evidence. (I have no alternative explanation, but I am unconvinced from a long interest in Forteana that the waterspout one holds water.) 87.81.230.195 (talk) 22:10, 11 March 2010 (UTC)[reply]
Interesting. I was reading Kafka on the Shore and I couldn't explain why the author put in so much supernatural occurrances in the story. Maybe Murakami was inspired by Fort. It reminds me of Magnolia where it was raining frogs in the end. I concluded that coincidents and interconnectedness among characters were so great that the rain of frogs became plausible and remind us anything was possible. Back to falling fish in Australia, the incident occurred hundreds of miles from any river or ocean btw. --Kvasir (talk) 05:54, 12 March 2010 (UTC)[reply]
The smaller the object the farther it can be carried by an updraft. Sand grains are carried all the way across the Atlantic Ocean from the Sahara Desert to the Carribean. Small fish can probably be carried hundreds of miles, but a great white shark ? Not so much. StuRat (talk) 19:18, 12 March 2010 (UTC)[reply]
Why do birds fall out of the sky every time you walk by? Just like me, they long to be close to you. Deor (talk) 21:33, 11 March 2010 (UTC)[reply]
Could perturbations in the flow of the Gulf Stream have to do with it? ~AH1(TCU) 00:24, 14 March 2010 (UTC)[reply]

Pressure in a liquid

Imagine a small particle inside a liquid. The pressure of the liquid will exert forces on it. There will be a downward force caused by the weight of the liquid above it, and an upward force caused by the water below the particle resisting the weight of the water above it, both forces equal in magnitude. My question is, are there any horizontal forces? I would imagine that the answer is yes, but I can't think of any way that the forces would arise... 173.179.59.66 (talk) 14:31, 11 March 2010 (UTC)[reply]

Yes, if the particle is floating in equilibrium, then the forces on all sides are equal. To help visualize this, think of a hole halfway up a can full of water: a stream is pushed out by the horizontal forces. Now think of two bricks with wet mortar in between. In gets squeezed out, right ? Well, think of the water above the hole in the can as one "brick" and the water below the hole as another. StuRat (talk) 14:39, 11 March 2010 (UTC)[reply]
That's exactly the explanation I'd give - but now I think about it, how do two exactly opposing forces (up and down) produce a force at 90 degrees to both? Aaadddaaammm (talk) 14:55, 11 March 2010 (UTC)[reply]
The stuff you are squeezing is a fluid (lots of small mobile particles), not a single solid. Consider if you have two spheres stacked but not quite perfectly aligned. As top one falls, it falls off to the side because the force (gravity, straight down) is acting a little sideways rather than straight at the bottom one. Another way to think is that the particles are all in constant motion in every direction. As the side comes closer, they cannot move that way but up/down is still available. And they may also bounce at an angle off the side, continuing to move up/down but coming back to center left/right. Fluid statics might have some info. DMacks (talk) 17:36, 11 March 2010 (UTC)[reply]

Boiling water

When boiling water, I've seen small bubbles form, sticking to the side of the pan. The same thing happens (sometimes) when I leave a glass of water sit for a while. Why is this? 173.179.59.66 (talk) 14:56, 11 March 2010 (UTC)[reply]

They are formed from the dissolved atmospheric gases coming out of solution. That’s why once formed they don’t grow any more. I.e., More oxygen and nitrogen can dissolve in cold water than warm at standard atmospheric pressure.--Aspro (talk) 15:01, 11 March 2010 (UTC)[reply]
see also nucleation157.193.173.205 (talk) 15:59, 11 March 2010 (UTC)[reply]

Is it true that freezing recently boiled water gives very clear ice cubes, for the reason Aspro mentions? --Dweller (talk) 15:58, 11 March 2010 (UTC)[reply]

Yes, boiled water does give clearer cubes for this reason; boiled distilled water is even better. In the days before domestic freezers, the companies that made large blocks of ice (which they then sold for people to put in their cool-boxes) kept ice clear by placing a tube in the center of the freezing can and bubbled air slowly through the water. This kept the dissolved minerals in constant circulation. Because it is the minerals and other foreign particles in water that serve as nucleation sites, the bubbles that did form, just floated up to the surface and burst. --Aspro (talk) 16:43, 11 March 2010 (UTC)[reply]
Did they want the ice clear just for cosmetic/marketing reasons? --Sean 13:13, 12 March 2010 (UTC)[reply]
A. I have no other reason to think that clear ice just looked nicer and gave the impression of being ‘pure’. Once one company did it I suppose the others had to follow suit. It guess this was not done for bulk ice used for railroad freezer wagons (for transporting meat). These used tanks of brine at each end and a fan driven off the axil to circulate the cold air within the wagon. Cloudy ice would not have mattered one iota. --Aspro (talk) 15:10, 12 March 2010 (UTC)[reply]


Okay, is there a physical explanation to why cold water holds more gas than warm water? 173.179.59.66 (talk) 16:11, 11 March 2010 (UTC)[reply]

This explanation is probably incomplete, but the basic reason is that warm water doesn't trap gas well. When warm, the water molecules are moving around more, with less static positions. The more energetic gas molecules have an easier time escaping under these circumstances. Cold water moves less, and is more organized in structure, so any gas that gets trapped has a harder time leaving. In both cases, the air molecules are occasionally dissolving, the only difference is that they are released more quickly by warm water, while cold water traps and holds them. —ShadowRanger (talk|stalk) 16:30, 11 March 2010 (UTC)[reply]
Cool, thanks. While we're on it, is there a significant pressure dependancy on a liquid's ability to trap gasses? 173.179.59.66 (talk) 17:20, 11 March 2010 (UTC)[reply]
Well, if the gas is under pressure, then it will dissolve more rapidly, and the final equilibrium will have more dissolved gas than water of the same temperature would have under lower pressure. In all cases it's a balance; how much gas is going in vs. how much is going out. —ShadowRanger (talk|stalk) 17:24, 11 March 2010 (UTC)[reply]
This sounds like some great potential content (if we can WP:V to get to the real underlying reasons) for Solubility#Factors affecting solubility. DMacks (talk) 17:27, 11 March 2010 (UTC)[reply]
I can't let a gas-coming-out-of-solution thread go by without linking to the fascinating Lake Nyos disaster. --Sean 13:17, 12 March 2010 (UTC)[reply]
On the subject of water, ice and important gases trapped therein: methane hydrate.--Aspro (talk) 15:16, 12 March 2010 (UTC)[reply]

Temperature of Diet Coke

I like Diet Coke. I normally drink it from plastic 2 litre bottles. Sometimes from 330ml cans. Occasionally (very occasionally) from those dinky glass bottles you see in American films and soaps, but are rarely sold here.

Anyway, it's my perception that Coke from glass bottles and cans is considerably colder when I drink it than it is from the plastic bottles. This, of course, all from the same fridge and the same position in the fridge.

Is this a flaw in my perception or is there some reason why it may be true? I had considered that it might be the volume of the container, not just its material that's the issue, but hey, you're the Scientists. --Dweller (talk) 15:08, 11 March 2010 (UTC)[reply]

Have you tried transfer some from each container into a couple of the same type of drinking vessels and then judging? It might be that your lips on the lower heat conductive bottle is affecting your perception.--Aspro (talk) 15:17, 11 March 2010 (UTC)[reply]
Interesting. I don't drink directly from the plastic bottle, but often drink direct from the can. So, I should pour from a plastic bottle into a chilled empty can, for a rigorous comparison? --Dweller (talk) 15:39, 11 March 2010 (UTC)[reply]
Maybe ask a friend to pour some coke from a chilled can into one glass and some from a chilled plastic bottle into another identical glass. Then you taste each and compare. I think that would be rigorous enough. Zain Ebrahim (talk) 15:43, 11 March 2010 (UTC)[reply]
I ask a friend to pour some

Coke from a chilled can into one glass and some Pepsi from a chilled can into another identical glass.

Verdict - Chilled Coke taste better to me.

and again

Coke from a not chilled can into one glass and some Pepsi from a not chilled can into another identical glass.

Verdict - Not chilled Pepsi taste better to me.

Conclusion - Chilled Coke and not chilled Pepsi.

manic

Would it be double blind if the friend marked which glass was which (say a sticker with #1 for the can an #2 for the bottle or vv) and left the room before you entered? Googlemeister (talk) 15:46, 11 March 2010 (UTC)[reply]
If you look at the three-dimensional clay model of the sensory homunculus ( just over half way down on this page), you will see that the lips contribute a great deal of the sensory information and so could over rule neural pathways in other parts of mouth and oesophagus. [15] This is why holding a piece of pair under the nose of someone who is blindfolded and whist giving them a slice of apple to eat. They will report that they are an eating pair because smell tends to over rules the sense of taste. (do they do these types of experiments in skools any more?)--Aspro (talk) 15:54, 11 March 2010 (UTC)[reply]
Here's a link to just the pic: homunculi.jpg. StuRat (talk) 18:12, 11 March 2010 (UTC)[reply]
All this talk about double-blind tests ... what's wrong with using a thermometer?157.193.173.205 (talk) 15:58, 11 March 2010 (UTC)[reply]
I thought someone would ask me that. Thermometers don't get on with me. I've never had a thermometer for home use that was both readable (mercury thermometers for me are the Emperor's New Clothes; so are Magic Eye pictures.) and reliable (the electric-stick-in-your-ear one we recently bought at great expense and fanfare resolutely refuses to give the same reading twice. We tend to take an average of three). --Dweller (talk) 16:02, 11 March 2010 (UTC)[reply]
I can confirm at least that Magic Eye pictures do "work". They aren't that exciting, but you really can, with some effort and unpleasantness, see realistically 3D images in them. Woooo. No comment on thermometers. --Mr.98 (talk) 01:58, 13 March 2010 (UTC)[reply]
You are comparing drinking from a chilled glass bottle or chilled can with drinking from a glass at room temperature, and you wonder why one feels colder than the other? Perhaps it is because the can is chilled but the glass is not? --Normansmithy (talk) 16:07, 11 March 2010 (UTC)[reply]
Okeydokey, I'll chill the glass and see. Thank you. --Dweller (talk) 16:12, 11 March 2010 (UTC)[reply]

Let see if I can guess the next question: Help! How do you un-stick a lip that's got frozen to a glass?!!!--Aspro (talk) 16:17, 11 March 2010 (UTC)[reply]

Well? !! What’s the answer Dweller? Or have you suffered a brown out?--Aspro (talk) 18:54, 11 March 2010 (UTC)[reply]
I have a feeling I must be missing something here or this is too obvious. Glass and metal have much, much faster thermal transfer rates than everyday plastics. When you touch the plastic lip of the 2L bottle to your mouth very little heat will be lost from your body. On the other hand, a metal or glass container will act as a kind of heat sink and provide you with a much cooler mouth feel. 64.235.97.146 (talk) 19:30, 11 March 2010 (UTC)[reply]
Yes you hav emissed something: the first reply!”It might be that your lips on the lower heat conductive bottle is affecting your perception.”--Aspro (talk) 21:16, 11 March 2010 (UTC)[reply]
If you don't have a scientific measure of the actual temperature of the liquid then it's likely that what's happening is that those very large bottles are getting cold on the outside and (let's say) for about an inch into the liquid - but that the majority of the liquid in the center simply isn't getting cold by the time you take it out of the fridge to drink it. The cans and bottles not only have smaller amounts of thickness of liquid for the fridge to chill - but also they both conduct heat better than plastic, so the contents get cool faster for that reason too. The way to test this supposition is to try drinking from one of the small 16oz plastic bottles that coke sometimes comes in. In that way you'd be controlling all aspects of the experiment except the volume of the container. SteveBaker (talk) 00:45, 12 March 2010 (UTC)[reply]


someone above said "Maybe ask a friend to pour some coke from a chilled can into one glass and some from a chilled plastic bottle into another identical glass. Then you taste each and compare." saying that is rigorous enough. ahahahahahahaha. So hilarious. The real answer is just buy a fast eleoctronic thermometer, and alternatively put it into the two two vessels, and see if the temperature is different. The above was cute though :) 82.113.106.93 (talk) 13:23, 12 March 2010 (UTC)[reply]

The premise is that the temperature is in fact the same, but the human perception of the temperature differs. The above advice could be used to determine if it is the perception only, or if the actual temperature that differs. Googlemeister (talk) 14:00, 15 March 2010 (UTC)[reply]

Fermi problem: neurons vs. transistors

So, I was thinking the other day about the sheer number of transistors that a successfully replicating yearly (via their biological symbiotic species, humanity), and I got to wondering: are transistors the fastest-growing species of life on Earth? Certainly they are being pretty successful. But a professor recommended comparing like-for-(relatively)-like. Thus, I'd like to know if anyone could help estimate the relative numbers of 'operational' neurons vs. 'operational' (or still capable of operation) transistors on planet Earth.

I have no idea where to begin answering this question, but trust some of you might be able to Fermiment some figures.

Thanks, 130.209.241.193 (talk) 15:12, 11 March 2010 (UTC)[reply]

Hmm. I think your prof. has sent you off on a tangent, the successful answer of which, will add little to the sum of human understanding. --Aspro (talk) 15:28, 11 March 2010 (UTC)[reply]
This page from 2006[16] suggests 10^19 transistors were made per year in 2006, although this has probably increased by a few orders of magnitude. You could probably extrapolate. You could try and count the number of microprocessors and estimate how many transistors they are likely to have, but remember that far more microprocessors are in embedded applications than in personal computers (on the other hand a simple microcontroller may have 10^5 while a new Intel processor may have 10^9), and don't forget RAM. Neuron will tell you there are about 10^11 neurons in the brain, and world population has lots of statistics. --Normansmithy (talk) 16:22, 11 March 2010 (UTC)[reply]
If you want to extrapolate figures for the present day, Moore's Law will help you calculate the increase in transistors per device, and you can multiply this by the growth in the number sold. I'm sure you can guess rough death rates for transistors and neurons. --Normansmithy (talk) 16:31, 11 March 2010 (UTC)[reply]
You'd have to squint pretty hard to see transistors as a life form. They're being made by humans, not other transistors. Paul Stansifer 17:53, 11 March 2010 (UTC)[reply]
I have to agree with Mr. Stansifer above. I'm not really sure if there is a meaningful question here. Transistors are not a life form by any definition of which I am aware. Certainly they do not grow, reproduce, consume raw materials, produce localized decreases in entropy, nothing. I am quite open to the idea of artificial life, but a minor electrical component is not it, in my opinion. Nor is a neuron a species either, though it is alive. I suggest reframing your question, if possible. — Knowledge Seeker 19:10, 11 March 2010 (UTC)[reply]
Ditto; ditto etc., ... Good scientists (IMO) have been those who have been able to identify the questions that are worth answering. Might I suggest, you suggest to your prof. to “post here, ” his reason why you should waist ‘spend your limited time’ on this? Just take a step back and think... there are billions of more important avenues of inquiry and speculation. Think your prof. is doing you a disservice to encourage you to travel down this route, a route which yields no meaningful insight to anything. It can only produce a non-sequitur.--Aspro (talk) 19:39, 11 March 2010 (UTC)[reply]

surely this would make hydrogen the most successful lifeform? Hydrogen obviously isn't life, but nor are transistors. They exhibit none of the characteristics listed here.--92.251.227.109 (talk) 19:42, 11 March 2010 (UTC)[reply]

It's weak - but arguable. Transistors cause humans to make more transistors. That's more like a virus than a proper "living thing" - and their descriptions are held as memes rather than genes...but they spread in similar ways. Person A buys an iPhone, person B sees it and becomes infected by the meme - causing him to buy one and thereby cause another iPhone to be manufactured. Eventually, people become immune either because they already had one or because their minds become immune to an old, tired meme. The iPhone evolves into the iPhone-II. Survival of the fittest applies - so poor phone designs die and good ones reproduce. Yeah - it's a hell of a stretch.
But as of a few years ago, I recall that someone had figured out that the total amount of memory devices ever manufactured was not equal to the storage capacity of a single human brain. So even if you could consider them to be in some manner like a lifeform - they are horribly outnumbered. SteveBaker (talk) 20:14, 11 March 2010 (UTC)[reply]
On that note, prions and definition of life may be of interest. Fire, tropical cyclones, and even the Universe may potentially be considered forms of life based on its current definition. ~AH1(TCU)

Free Particle

The free particle article only dealt with a particle of well defined momentum, so I thought I'd clarify a few things here, and get them added to the article, however I ran into problems with my understanding of the maths. This is quite elementary QM so I am a bit concerned by how trouble some it has been.

I guess this is a little bit of spot the mistake.

For well defined momentum, the momentum , allowing us to define a propogation operator where defined by the schrodinger equation.

thus

since

therefore

given that the particle is initially localised at , therefore

performing first the integral over p

and then x

thus the probability

the probability distribution is independent of , and infinite everwhere at despite the boundary condition that it was localised at . Help! —Preceding unsigned comment added by 129.67.39.49 (talk) 16:10, 11 March 2010 (UTC)[reply]

Your inital state wave function is not normalized correctly, I don't think.Dauto (talk) 19:28, 11 March 2010 (UTC)[reply]
One thing that jumps out is that rather than 1. Rckrone (talk) 19:37, 11 March 2010 (UTC)[reply]
True, for some reason when writing I was thinking of , I have corrected the mistake.
I'm not sure if the math is right here or not, but it's worth thinking about what result you expect to get for this problem. If you start out with a particle truly localized at a point, then it's evenly distributed across the whole momentum space (it can't really be represented by a distribution at all). After any non-zero time t, the distance it would move is proportional to the momentum, so it would be evenly distributed across the whole position space (again not really representable with a distribution), so I don't know if you can expect a sensible answer. Rckrone (talk) 22:38, 11 March 2010 (UTC)[reply]
I think the math is fine. I get the same result. The classical analogue is infinitely many particles flying apart from a point with momenta evenly distributed in momentum space. The density falls off like 1/t. Total energy (or probability) is "conserved" inasmuch as it's always infinite. This also shows up in the low-density nonrelativistic limit of big bang cosmology (with the same 1/t falloff). To get nice time evolution you need to start with a nice normalizable wave packet, as in the example at the end of Wave packet#Mathematics of wave packets. -- BenRG (talk) 23:03, 11 March 2010 (UTC)[reply]
I haven't checked the math, but the result seems plausible. If you recall the Heisenberg uncertainty principle, no particle can have both definite momentum and definite position. If you insist that the particle is truly in a momentum eigenstate, then it must have equal and uniform probability of having any possible position. Or in the limit of truly infinite space, the "normalized" wavefunction goes to zero everywhere. Dragons flight (talk) 01:16, 12 March 2010 (UTC)[reply]

Ok, in that case can you help me understand the interpretation. I thought that due to uncertainty, asserting that at t=0 the particle is localized meant that by definition in p-basis the wavefunction included a non-zero probability for all possible momenta. So that any result in x-basis is allowed by heisenberg uncertainty (since nothing is known about the momentum). And thus I would have expected the delta function to spread like a guassian growing in width over time; as for a given x, the number of momentum eigenstates that permits the particle to have reached x grows with t. Also this fits with the recquirment that probability flows continuously; which the answer derived above does not seem to. —Preceding unsigned comment added by 129.67.39.49 (talk) 03:35, 12 March 2010 (UTC)[reply]

Your solution is essentially correct for all times except . To see why that happens you should use a gaussian as intial condition instead of the delta function.
.
That initial condition will converge to the delta function you used when the limit is taken, except for the extra normalization factor that was missing from your solution (as a mentioned on my earlier coment). That factor cancels the infinity you found on your solution. If you use that initial condition and do a calculation similar to the one you've done before, you get
When the limit is taken the expression above coverges to your solution (times the extra normalization factor) for all times except when it converges to the initial condition. Dauto (talk) 16:29, 12 March 2010 (UTC)[reply]
What about the expanding gaussian? If you want to see the expanding gaussian you must keep second order terms in the expansion of the exponential argument.
And we can now get the expression for the probability
Which has the gaussian you were looking for. Dauto (talk) 03:32, 13 March 2010 (UTC)[reply]

Too much Potassium in diet

what are the negative effects of having too much potassium in one's diet? Also, are there are any negative effects of having too much chlorine in one's diet?--92.251.227.109 (talk) 19:34, 11 March 2010 (UTC)[reply]

Not sure about too much potassium in the diet, but Wikipedia has an article on Hyperkalemia, which is too much potassium in the blood. Does that article help? Zain Ebrahim (talk) 19:43, 11 March 2010 (UTC)[reply]
See Potassium_chloride#Biological_and_medical_properties. --NorwegianBlue talk 19:44, 11 March 2010 (UTC)[reply]
I already read that article, but it only talks about overdoses, not simply consuming more than you should.--92.251.227.109 (talk) 19:48, 11 March 2010 (UTC)[reply]
Chlorine in the diet is always in the form of compounds ( chlorine is too reactive to exist in it’s atomic form). Most of them are better avoided ( I avoid over generalizing by calling them all toxic). I think ( off the top off my head0 that too much potassium will just be pissed out. Although pottasium nitrate ( for curing bacon etc.,) has been linked tio cancer .--Aspro (talk) 19:47, 11 March 2010 (UTC)[reply]
Ah right that's very helpful thanks! What would happen to someone with kidney failure who could not urinate, who took in a lot of potassium?--92.251.227.109 (talk) 19:48, 11 March 2010 (UTC)[reply]
That is indeed a problem. They try to limit their potassium intake, and also may use potassium binders, such as sodium polystyrene sulfonate. See Sodium_polystyrene_sulfonate#Medical_use. StuRat (talk) 19:55, 11 March 2010 (UTC)[reply]
If they can’t piss it out: Their potassium/sodium balance would be servilely compromised -is the short answer to that . Can you expand on that. Remember: we don’t give medical advice. --Aspro (talk) 19:58, 11 March 2010 (UTC)[reply]
Have you read Renal_failure yet? --Aspro (talk) 20:00, 11 March 2010 (UTC)[reply]
I hear Obama is trying to get something through congress to help people with medical problems the same way as here in Europe. Do you have to ask these questions because your American?--Aspro (talk) 20:06, 11 March 2010 (UTC)[reply]
I'm not American I don't have kidney problems. I just dined at a friend's who used a 1:1 mix of potassium chloride and sodium chloride instead of regular sodium chloride table salt and I got curious. Also I have my own opinions of government run healthcare services after some stuff the Irish government pulled (forced every insurer to pay a state-owned insurer money, forcing some companies out of the market).--92.251.227.109 (talk) 20:10, 11 March 2010 (UTC)[reply]
Here in the UK we have products under the such names as “Lo Salt” with 60% or less sodium. Remembering what I said about medical advice: Unless one has kidney, heart or diabetes problems, this modified form of salt should be OK. Even then, it depends on the individual case. In my view it is better just not to see a doctor in the first place-- ignorants is bliss- but from a doctor’s point of view “this behaviour is not very profitable” Large amounts of potassium salts will also stop the heart but no normal person will want to drink this amount. It ( the cardiac muscle) will just not be able to re-polarise – but that’s for another time. --Aspro (talk) 20:29, 11 March 2010 (UTC)[reply]
Erhummm.., I have just re-read that post above...
”I just dined at a friend's”
One can just imagine it.....”Heard you had 92.251.227.109 round for dinner last night... AND WHAT did he think of your culinary skills?
Hmm well! He thought my choice of table salt very interesting! Ho! Ho! Ho! --Aspro (talk) 20:54, 11 March 2010 (UTC)[reply]

I actually attended a lecture at Columbia University Medical Center recently given by nephrologist Dr. Len Stern, who spoke a great deal about the terrible effects of high potassium diets. He showed a graph of what is termed an "ideal protein diet" and how it results in a much greater potassium intake than is ideal. So, while I cannot describe the effects of such a diet and cannot comment on particulars, you may take the initiative and find Dr. Stern's email address from Columbia's website and ask him for the details. DRosenbach (Talk | Contribs) 00:47, 12 March 2010 (UTC)[reply]

Going by the amount of potassium found on food package nutrition data labels about 3.5 grams of potassium is required with a 2,000 calorie diet per day or about 1.2 grams per a 1 in 3 meal plan per day. 1.2 grams of potassium chloride when weighed out looks like a quarter teaspoon. since both potassium and chloride are part of your electrolytes they are required by your body. As a personal note when I eat foods that are low in potassium according to this recommended amount I get chest pain. When I add enough potassium to bring my meals up to the recommended amount the chest pains not only go away but I sleep like a rock. I'm also careful to be sure the potassium is completely dissolved and spread throughout the meals such as by dissolving it in water and then using the water to cook or as a solvent or even to stay hydrated throughout the day. This last item is important because potassium unlike sodium will produce lesions in tissue instead of helping lesions heal like sodium. 71.100.11.118 (talk) 02:40, 12 March 2010 (UTC)[reply]

Chemical compounds with many elements

What is the chemical compound with the most chemical elements inside it? --84.61.135.112 (talk) 20:34, 11 March 2010 (UTC)[reply]

It's probably not near the highest, but DNA must have quite a few. StuRat (talk) 20:44, 11 March 2010 (UTC)[reply]
DNA is a very complicated molecule, but it doesn't actually have too many elements in it. I think is just has Nitrogen, Oxygen, Hydrogen, Phosphorus, and, of course, Carbon. Maybe it has some sulfur in it too, but it only has about 5 or 6 elements at the most. --The High Fin Sperm Whale 20:50, 11 March 2010 (UTC)[reply]
I assume you mean distinct elements, as opposed to "atoms per molecule"? DNA definitely wins on the latter, but probably not the former. Nevertheless, I suspect a carbon-based molecule would win for overall diversity, since carbon chains allow for a near infinite number of attachment points. You'd need to construct it artificially, but you could theoretically stick most of the (non-noble) periodic table on a long enough carbon chain. —ShadowRanger (talk|stalk) 20:55, 11 March 2010 (UTC)[reply]
Fluoride is a very reactive halogen and will encapsulate most compounds . Is this the the tree your barking up?--Aspro (talk) 20:58, 11 March 2010 (UTC)[reply]
Of the top of my head it’s most things from Hydrogen to Uranium.--Aspro (talk) 21:01, 11 March 2010 (UTC)[reply]
If metal alloys are valid answers, one of those? Googlemeister (talk) 21:02, 11 March 2010 (UTC)[reply]

Here is a molecule (CAS #289501-11-7) with 9 different elements: C33 H40 Cl N O P S Si . I name: Phosphonium, [(3S,4Z)-4-chloro-3-[[(1,1-dimethylethyl)dimethylsilyl]oxy]-5-(2-methyl-4-thiazolyl)-4-penten-1-yl]triphenyl-, iodide (1:1) 24.150.18.30 (talk) 02:17, 12 March 2010 (UTC)[reply]

And another (CAS # 807332-34-9), with 10: C76 H62 Br2 Cu2 N4 P4 Pt S2 . 2 Cl O4 Platinum(2+), bis[(5-bromo-1,10-phenanthroline-κN1,κN10)copper][μ3-[1,2-ethanedithiolato(2-)-κS:κS,κS':κS']]bis[μ-[methylenebis[diphenylphosphine-κP]]]-, diperchlorate (9CI)

I don't think that you will find one with much more than 10, but I'm also not sure how you would do an exhaustive search for it.24.150.18.30 (talk) 02:17, 12 March 2010 (UTC)[reply]

It depends on how you define a compound. There are many minerals, for example, with astoundingly complex crystal structures. Some varieties of Tourmaline for example have up to 15 elements. --Jayron32 04:37, 12 March 2010 (UTC)[reply]

I searched a crystallography database and the highest number of elements in one compound was 22: Johnsenite-(Ce) of the Eudialyte group and "Ca2(Y1.53Dy0.10Er0.09Yb0.08Gd0.07Ce0.02Tb0.02Fe0.02La0.01Nd0.01Sm0.01(PrEu)0.01Ho0.01Tm0.01Lu0.01)((Si3.87Al0.13)O12)CO3·H2O" from "G. Giuseppetti C. Tadini M. Oddone, Neues Jahrbuch für Mineralogie. Monatshefte, (1989) p153". There were about a thousand hits with more than 10 elements, and 50 hits with more than 15 elements.

These things sort of push the traditional definition of 'chemical compound' a bit, since they often have flexibility in the proportions of certain elements that are chemically and/or structurally similar.

As illustrated by some of the responses above, there are many "proper" compounds without such flexibility (non-stoichiometry), especially in coordination complexes with organic ligands. These combine the huge variety of carbon-based compounds with metal atoms that can have many different atoms bonded to them simultaneously - often six, sometimes more.

Ben (talk) 15:28, 12 March 2010 (UTC)[reply]

Here's the complex with the most different elements that I could find on the Cambridge Structural Database: Eur. J. Inorg. Chem. 2007, 2721–2728: C80H103B2Cl16Co4CsF8N12NiORu3S3.
Ben (talk) 15:34, 12 March 2010 (UTC)[reply]
Organic compounds probably tend to have fewer elements, as Methionylthreonylthreonylglutaminylarginyltyrosyl...glutaminylglutaminylserxisoleucine (C169723H270464N45688O52243S912) contains many atoms but only five individual elements. Also, how can an element have a decimal subscript for their atoms, or is it similar to the empirical hypothesis that water could be H1.5O? ~AH1(TCU) 23:54, 13 March 2010 (UTC)[reply]

Addicts - do they prefer to be cured?

I've never been addicted to anything, except perhaps tea. Would an addict, such as an alcoholic, like to be perfectly cured and lose all interest in or cravings for alcohol? Or is their desire for alcohol so strong that they would prefer to stay an alcoholic even if a perfect cure is available? 78.146.0.232 (talk) 21:40, 11 March 2010 (UTC)[reply]

Before or after they are cured? Depending on the alcoholic, they may or may not be aware of the consequences of their alcoholism. And they may or may not have a reason for drinking; in many cases it's a form of self-medication for depression. If they are aware of the damage they are causing and lack an ongoing reason to drink, then clearly then most of them would presumably welcome a cure that removed all physiological and psychological cravings. On the other end of the spectrum, they might not, or they might return to alcohol even after the cure is applied, as the ongoing reasons for self-medicating have not been addressed. —ShadowRanger (talk|stalk) 21:45, 11 March 2010 (UTC)[reply]
Elaborating: By and large, people don't prefer being addicted. But if they have reason to quit, the pull of alcohol isn't overriding their free will all the time, or even for particularly long periods. The problem is that in normal circumstances, staying sober means the alcoholic must maintain perfect control at all times; one slip up can undo all their work. If a cure could be devised to immediately remove dependence, that would make it *much* easier to stay sober. —ShadowRanger (talk|stalk) 21:48, 11 March 2010 (UTC)[reply]
As an example of someone who might not take the cure: I know someone (details unspecified for privacy reasons, I'll use singular "they" to describe) who drinks about a liter of wine a day (down from two liters, which had been the usual amount for decades). I doubt they have been fully sober for more than an hour a day in over a decade. But they don't see any consequences from their actions, as their significant other is an enabler who denies there is a problem, and their siblings drink more than they do. On top of that, they grew up a victim of sexual abuse and have never come to terms with it. The wine is likely treating depression resulting from the abuse, and the consequences of the drinking are relatively minor (today, though the decreased inhibition led to some awful behavior when they were younger). Since they don't see a problem, and the drinking is treating an underlying condition, they might resist a cure. —ShadowRanger (talk|stalk) 21:54, 11 March 2010 (UTC)[reply]
Is there more alchohol in a liter of wine, or a 6 pack of beer? Googlemeister (talk) 22:02, 11 March 2010 (UTC)[reply]
Depends on the wine and beer, but generally wine is something like 11.5% alcohol and beer is something like 4%. A standard beer can is 12 oz of liquid, thus a six pack is some 2.88 oz of alcohol. A liter of wine (33.8 oz of liquid) is 3.89 oz of alcohol. So the wine wins in the alcohol content contest. But again, answers may vary depending on type of wine and beer, and amount of beer in the six pack. --Mr.98 (talk) 22:36, 11 March 2010 (UTC)[reply]
In the particular case in question, the wine of choice is a dry red wine, with 12.5-13% ABV IIRC. So the daily intake of alcohol is about 4.25 oz. One drink is usually considered to be about a half ounce of alcohol, so this is about 8.5 drinks. The reduction from 17 drinks per day to 8.5 does not appear to have affected the level of inebriation (low level, but noticeable, particularly into the early evening), so it's likely that the person in question's liver has been damaged such that metabolization of alcohol occurs more slowly. —ShadowRanger (talk|stalk) 23:32, 11 March 2010 (UTC)[reply]

The answer to your question is both. Yes they would like to be cured. Yes they have such a strong craving that they would like to continue to abuse it. That ambivalence is at the heart of any addiction when it has reached the point that the person recognizes it is hurting them. alteripse (talk) 12:10, 12 March 2010 (UTC)[reply]

Would your question apply only to substance addiction, or would it include addiction to non-substances such as Internet addiction? ~AH1(TCU) 02:54, 13 March 2010 (UTC)[reply]

Chimpanzees and humans, for example, are said to have 99% of their genes in common. Dodos and mammoths have some but not all of their genes available from dead specimens.

Would it be possible to replace the missing genetic material with material from a related living species such a a pigeon or elephant, and get a living creature as a result? If not, why not?

Is it currently technologically possible to synthesise a sequence of genes just from chemicals in a laboratory, put them in an egg, and get a living creature as a result? 78.146.0.232 (talk) 22:11, 11 March 2010 (UTC)[reply]

Well, you have two issues here. One is whether you can fill in missing spots of DNA. I am not particularly sure this is possible but I wouldn't put it totally beyond very clever computational geneticists who happened to be very lucky about what was missing. If it was something specific to the species, and you had no other samples, then I imagine you would be out of luck. If it were something common, then you are OK. The second is whether you can take that raw DNA and make a living creature out of it. At the moment, the answer is, "it's really hard," but in another fifty years or so it probably won't be so hard. The relevant article part seems to be Extinction#Cloning; see in particular the case of the Pyrenean Ibex, which was made un-extinct for a short amount of time, on the basis of one very good tissue sample. --Mr.98 (talk) 22:55, 11 March 2010 (UTC)[reply]
I've read some articles on genetic sequencing that claim an incredibly high success rate at reconstructing DNA from fragments. I believe similar proposals have been made to resurrect the Woolly Mammoth by filling in gaps in a reconstructed genome with DNA from an elephant, then replacing the DNA in an elephant embryo and bringing it to term in an elephant surrogate mother.[17][18] It's supposed to be possible, just extraordinarily expensive (Less plausible proposals involve using frozen mammoth sperm to impregnate a modern day elephant). Similarly, resurrecting Neandertals might be possible, but the ethical concerns make it highly unlikely to occur in my lifetime. A note: Dinosaurs died out far too long ago to leave DNA that could be reconstructed by the technique in question, so no, Jurassic Park won't be happening soon. —ShadowRanger (talk|stalk) 23:24, 11 March 2010 (UTC)[reply]
It's hard to believe that you'd have a problem here. Let's say that 99% of the DNA in your sample is destroyed. What that means is one of two things: EITHER...
  1. 99% of cells have ruptured or something and have no useful DNA inside and the remaining 1% of the cells are intact...so just find a good cell and then use Polymerase chain reaction to make the stuff by the bucketful...OR...
  2. Each cell only has 1% of it's DNA left intact...but if you take the DNA from a million cells, you can find any given stretch of the DNA in lots of different cells so all you need to do is to sequence the fragments and write some rather simple software to stitch it back into an intact strand.
I can't think of any but the most contrived set of circumstances in which you'd need to take DNA from another species. Just find another cell in the same creature - the odds of the same bit of DNA being missing from every single cell in your sample seems astronomically unlikely. The only way I could imagine there being "gaps" would be if a spectacularly large amount of damage had been done and the sample size was microscopic. SteveBaker (talk) 00:31, 12 March 2010 (UTC)[reply]
The sample size may indeed be microscopic, as in the Jurassic Park scenario, where the blood in the gut of a mosquito preserved in amber is the source. And, while dino DNA can't survive by being frozen like a mammoth's, that doesn't mean there's no way it could survive. Here's one possible case: [19], [20]. StuRat (talk) 00:54, 12 March 2010 (UTC)[reply]
The problem with that idea is that you have no idea what the DNA you find there is from. You could go to a lot of trouble and wind up resurrecting a Jurassic lizard or a frog or something! You'd have no idea how to turn the DNA back into a living animal because we have no concept of how to deduce what kind of animal you'd get from what kind of DNA - so you wouldn't have a way to know what kind of cell to implant it into - or what kind of animal to provide a surrogate womb. I suppose if the DNA was sufficiently similar to (say) a modern bird then you might maybe be able to guess that you have a dinosaur of some kind - but if it's similar to a modern reptile - then you might just have DNA from a Jurassic Gecko. Given the ratio of numbers of large dinosaurs versus small animals like frogs - the probability of that super-rare-mosquito-in-amber-stomach-contents being blood from a T-Rex are absolutely astronomically small. SteveBaker (talk) 16:02, 13 March 2010 (UTC)[reply]
While a T-Rex would be wonderful, I'd still think many biologists would consider a resurrected Jurassic frog to be quite a coup. Also, by the time the technology is at such a state that we could do this, I'd expect the reconstructed DNA genome to tell us fairly precisely what type of animal we have. StuRat (talk) 03:24, 14 March 2010 (UTC)[reply]
I see a couple problems with the idea of filling in the gaps of an extinct species with another:
1) If you have gaps in the extinct species, then how do you know that the missing DNA is identical to the donor species ?
2) If the species of interest has badly degraded DNA, you may only have tiny fragments left, maybe only a base pair or two each. With such small strands, it may be impossible to determine where each fits into the sequence, and thus to figure out where the gaps are. Just like decoding a cypher, you need sequences of a certain length to even begin the process. StuRat (talk) 01:18, 12 March 2010 (UTC)[reply]
The severity of Stu's first objection depends on whether the goal is a perfect copy of the extinct animal or a viable approximation. —Tamfang (talk) 22:06, 12 March 2010 (UTC)[reply]

It is indeed possible to synthesize DNA de novo; see our article on Mycoplasma laboratorium for the most ambitious project currently going. It might be conceivable to resurrect an extinct lifeform by inserting genes into a closely related living species, for example bringing back the passenger pigeon by editing the genes of an existing pigeon. For distant species it's almost certainly out of the question -- there are huge numbers of interactions between genes, and there are also commonly substantial differences in the non-coding DNA that lies in between the genes. Looie496 (talk) 22:34, 12 March 2010 (UTC)[reply]

A further problem, mentioned by Mr.98 above, is that the developement of a genome into a viable organism depends on its continuous and complex interaction with exactly the right actively co-operating environment (in the case of a mammal, its mother's womb), which provides exactly the right physical conditions, chemical substances and chemically (hormonal and enzymatic) cues at all times. This process frequently goes wrong even in normal circumstances, hence the occasional birth of deformed offspring and the frequent early death and reabsorbtion of embryos at an early stage (I believe that in humans this runs at around 30% of all pregnancies, most of which remain unsuspected by the mother). In the case of an extinct species sufficiently 'different' from an unextinct one as to be interesting, this environment is by definition not available, and it is very unlikely in most cases that any living species could approximate it closely enough to result in a viable organism, still less one that would not differ from the 'original' in phenotypically significant ways. 87.81.230.195 (talk) 03:56, 13 March 2010 (UTC)[reply]

March 12

Looking for a bird

Does anyone know of a complete list of species featured in The Life of Birds? In particular, I am looking for one bird species, it must have been in "Signals and Songs" (episode 6). Its song sounded like early electronic analogue synthesizers, or someone rapidly turning a ham radio dial. I think Attenborough's commentary mentioned that it had to learn to produce something like 40 different sounds in order to get its song right. I think it was North American, small, and dull-coloured (mostly brown). Thanks in advance! ---Sluzzelin talk 01:22, 12 March 2010 (UTC)[reply]

Well no doubt the bird with the strangest vocals in that series was the Lyre bird, that's the one that can imitate other birds and even things like camera shutters and chainsaws, but I don't know if anyone's brought a Moog for it to imitate:) . However it's not American, and its tail is a big give away, but I distinctly remember that in a lot of close shots with the tail mostly if not totally cropped out it looks small and dull colored. Vespine (talk) 04:24, 12 March 2010 (UTC)[reply]
Thanks, I too remember the supreme superb lyre bird distinctly, but as amazing as it is, it isn't it. The one I'm looking for isn't mentioned in the article. It didn't imitate. It just had its entirely own spacey sound. :-) ---Sluzzelin talk 04:31, 12 March 2010 (UTC)[reply]
I think I found it. This page mentions the cowbird using "40 different notes, some so high we can't hear them". Judging by the sound sample on this site, it's the Brown-headed Cowbird. My memory had something slightly more spectacular in storage, but this is definitely it. Thanks again. ---Sluzzelin talk 06:05, 12 March 2010 (UTC)[reply]
(I realize I was the one asking, but maybe others are interested too): The Brown-headed Cowbird article doesn't mention its song, but I found something under Lateralization of bird song. ---Sluzzelin talk 12:24, 12 March 2010 (UTC)[reply]

Reflex subwoofer query

I made a ported (reflex) sub-woofer with a vertically upward firing drive unit and a vertically downward firing port for bass instrument reproduction. I found that increasing the distance between the port and the floor (to about twice the ports diameter) seems to give a much richer sounding bass then when I used only a single diameter spacing (as recommended in some sources as the min). Why is this, and how does the spacing between port and floor affect the response/efficiency of the system? In particular, why do larger spacings seem to give more 'fruity' bass with apparently increased high frequency response?--79.76.188.14 (talk) 01:47, 12 March 2010 (UTC)[reply]

Sound pressure waves emitted from the port are reflected by the floor so creating a resonant column of air. Raising the woofer increases the height of the column so lowering its resonant frequency. That reduces the "boxyness" of the sound in the same way as would enlarging the cabinet. The sound frequencies are too low for a floor carpet to provide effective damping but you might notice some improvement from filling the whole space between the port and floor with a roll of carpet felt or foam. Cuddlyable3 (talk) 11:08, 12 March 2010 (UTC)[reply]
I find myself wondering whether more of the sound emitted by a downward facing speaker doesn't end up in the apartment below you than in your own. StuRat (talk) 12:31, 12 March 2010 (UTC) [reply]
Depends how stiff the floor is.--79.76.188.14 (talk) 20:27, 12 March 2010 (UTC)[reply]

Likely outcome of an all-out thermonuclear war between the US and Russia at the height of their nuclear capabilities?

US and USSR nuclear stockpiles during Cold War
One of many possible fallout scenarios from a mid-1980s attack on the US

Obviously no-one can 'win' a global nuclear war, as such - but I remember reading somewhere that in the event of a full-on nuclear exchange where everyone threw everything at everyone else, it was hypothesized by that it would likely be that Soviet Union that would 'continue to exist' in some form in the aftermath (mainly due to its greater land mass and more dispersed population), whilst the US (and all of Europe - West and East) would be almost completely destroyed. Am I remembering correctly here? Does anyone know the piece of research I am referring to? Does this sound at all plausible? Thanks. --95.148.106.148 (talk) 02:22, 12 March 2010 (UTC)[reply]

Just for reference, according to that infallible source, Wikipedia, the peak of the cumulative US-USSR nuclear stockpiles was in 1986, at 63,977 total warheads (23,254 US, 40,723 USSR). That would also be a period of considerable technological sophistication in delivery vehicles—ICBMs, SLBMs, MIRVs. So that's a lot of destruction.
The main problem here has and continues to be that calculating that kind of destruction is pretty hard. Most estimates probably under-estimate the destructive power of the weapons—they exclude good predictions of fallout and fire effects, or the long-term effects of kicking up that much (radioactive) dust into the atmosphere (which would probably have effects on the climate as well). Lynn Eden's book Whole World On Fire discusses this at some length. Certain weapons effects are "easy" to model—blast pressure, prompt neutron radiation, gamma rays, etc.—and some are quite hard—firestorms, fallout, etc. As a result, most calculations focus on the "easy" bits and exclude the hard bits, even though the hard bits actually do a huge amount of the damage (most of the destruction in Hiroshima and Nagasaki was caused by fire, not radiation).
But I think it is clear that if the US were hit by a large nuclear exchange, depending on the weather patterns, the fallout itself would be pretty destructive to its ability to continue as any kind of a nation. It would also surely lose a huge, huge percentage of its population, clustered as they are in big cities on the coasts. The USSR would probably do a little better, though I'm not sure it would be all that enviable. The interior would surely be targeted (as would the US's) because that's where its factories and silos were (as with the US; note the FEMA fallout map picture that shows how hit the midwest is for this reason). The resulting fallout would coat the interior areas pretty good. The lands would not be productive without massive cleanup and long-term birth defects and etc. would be rampant amongst any survivors not living far underground in mythical Strangelove-esque fallout shelters.
If you pick an earlier point in time, when the stockpiles are not so huge and the delivery mechanisms are not so good, then the USSR does a lot better—bombers are not going to saturate the interior so much. The US also probably does a lot better, as Soviet delivery mechanisms were really quite poor until the late 1950s. --Mr.98 (talk) 02:41, 12 March 2010 (UTC)[reply]
I think another major issue would be the reliability of the payload delivery systems. Many articles I have read over the last few years suggest that many, many Russian delivery systems and payloads themselves are faulty, and would likely not launch correctly, if at all. Beach drifter (talk) 03:55, 12 March 2010 (UTC)[reply]
Is this referring to now, or then? I know that the Russian nuclear arsenal has supposedly fallen into disrepair somewhat since the end of the Cold War. Thanks for the answers so far, by the way. It boggles the mind that things ever got to the point where people needed to sit and calculate this stuff and that there were people who would be prepared to actually carry it out (or put the choice to do so in the hands of computers!) - it seems completely insane, not to mention quite obscene. --95.148.106.148 (talk) 04:03, 12 March 2010 (UTC)[reply]
Many people, including me, disagree on your last point, but this is not a discussion board. 74.212.140.226 (talk) 07:08, 12 March 2010 (UTC)[reply]
To clarify: The objective of having those weapons was as a deterrence. In order to deter, it was 100% essential that the enemy be utterly convinced that you'd push the button. Hence, even if you had no intention of ever doing such a crazy thing - you had to convince them that you would. Thus, at least the public face had to be that you'd do it. Although it pains me to say it - it worked. There was no world war three. The Soviets didn't invade Europe - despite having conventional forces that were easily capable of doing so. The US didn't extend it's power throughout the world in a way that the Soviets would have found unacceptable. The American people got so sick of the idea of war that it became difficult for them to carry through with them - the Russians ran out of money. We got through that period with nothing more than a few minor skirmishes - and it was actually a fairly peaceful time for the participants. The push that the cold war gave to technology has given us cheap computers, satellites, GPS and the Internet to name but a few. I don't like it - but it did work exactly as advertised and it had significant positive outcomes. The problems the world has now relate to the fact that one side of the conflict regards Mutually assured destruction as an acceptable outcome so deterrence doesn't work. SteveBaker (talk) 13:47, 12 March 2010 (UTC)[reply]
It's of note though that there is a fair consensus that the massive build-ups led to situations of overkill, well above and beyond what was probably necessary for deterrence (and there is a lot of evidence that the Cold War politicians and military people never actually really accepted deterrence as their goal—many were explicitly interested in the possibility of a debilitating first strike and feared that the other side was pursuing it as well). And there is a debate about whether creating the realistic possibility of mega-deaths really was preferable to the Soviets running Western Europe. I think there are many who would disagree that all of the skirmishes in that period were "minor"... a few million people died on all sides in Korea, Vietnam, Afghanistan, etc. At virtually no point were the Cold War powers not engaged in some kind of costly warfare. The idea that the Cold War powers acted as logical agents here is a somewhat naive reading of the history. It is luck as much as anything else that prevented nuclear exchanges, and there were some very, very close calls. --Mr.98 (talk) 16:40, 12 March 2010 (UTC)[reply]
I wouldn't say the Russian delivery systems are faulty. Russia has a better track record than the USA developing reliable rockets. Quest09 (talk) 10:21, 12 March 2010 (UTC)[reply]
It's really pretty hard to know if the delivery systems would have worked on either side. The US in particular only did component testing (they fired exactly ONE live rocket with a live warhead on it in the entire Cold War—and even that wasn't a stockpile warhead, if I recall). The essential problem is that you can't test whether everything would work correctly without actually using it, which means death. This problem and its historical consequences is discussed in depth in Donald MacKenzie's Inventing Accuracy: A Historical Sociology of Nuclear Missile Guidance, which is a really interesting book if you care about such things! --Mr.98 (talk) 16:40, 12 March 2010 (UTC)[reply]
One thing to consider is that the *vast* majority of US nuclear strategy was focused on destroying the Russian nuclear force, and presumably the Russians had a similar idea. This means that a total exchange would be very unlikely as each successful hit by one side means that many fewer warheads for the other side to launch. --Sean 13:28, 12 March 2010 (UTC)[reply]
How do you figure? Unless you do something to prevent the weapons launch, there will be a warning of 15 minutes or so for central missile silos. More then ample time to launch the weps, so if you target enemies silos, unless you have a way of masking your hundreds of launches, you are not going to destroy very many opposing warheads. Googlemeister (talk) 14:02, 12 March 2010 (UTC)[reply]
Yes, I think your logic is wrong here. It's a "use 'em or lose 'em" scenario. There is enough time between launch and detection for the other side to launch theirs—that's why the big missile silos are out in the middle of the land masses (to give more time), and a considerable part of the force is kept in hard-to-detect submarines. That's why in the FEMA fallout diagram above, Montana, North Dakota, and Wyoming gets plastered so hard—they are assuming that in a full nuclear exchange, the Soviets would be seeking to really saturate the air force bases out there that were basing Peacekeeper and Minuteman missiles at the time. That does not necessarily mean at all that the Soviet hits would be successful in knocking out US capabilities—the missiles could easily pass each other on the way there. --Mr.98 (talk) 16:40, 12 March 2010 (UTC)[reply]

A strange game. The only winning move is not to play. How about a nice game of chess? Coreycubed (talk) 15:20, 12 March 2010 (UTC)[reply]

I don't understand this discussion. An all-out nuclear war would, most likely, result in Nuclear winter, which would wipe out most of the world. Whether the US or USSR would fare better is completely irrelevant (after the first month or two). A 2007 study (using modern climate modelling techniques, which are pretty reliable) predicted that if all of the current stockpiles were launched, which are about 1/3 the size of the peaks, then the results would be:
A global average surface cooling of –7°C to –8°C persists for years, and after a decade the cooling is still –4°C (Fig. 2). Considering that the global average cooling at the depth of the last ice age 18,000 yr ago was about –5°C, this would be a climate change unprecedented in speed and amplitude in the history of the human race. The temperature changes are largest over land ... Cooling of more than –20°C occurs over large areas of North America and of more than –30°C over much of Eurasia, including all agricultural regions.
Those kind of drops would leave North America and Eurasia in Arctic conditions, there would be no significant agriculture. I haven't found estimates of death toll, but I would personally guess that we're talking at least 90% of the world's population being killed before the climate started to get back to normality (and that is assuming there isn't a Snowball Earth scenario where you get a runaway cooling which leads to thousands of years of global ice). --Tango (talk) 17:21, 12 March 2010 (UTC)[reply]
Assuming it is a USA vs. USSR war, the Southern Hemisphere would probably do much better than the North with respect to nuclear winter. Stratospheric wind patterns make it hard for dust injected in one hemisphere to cross the equator and reach the the other hemisphere (as shown by high latitude volcanic eruptions). Dragons flight (talk) 17:39, 12 March 2010 (UTC)[reply]
Contrary to the 1957 nuke fiction On the Beach (novel). In his book for some reason the fallout radiation levels do not decay to a very low level in a couple of weeks as most sources predict, but continue lethal for years. Edison (talk) 18:09, 12 March 2010 (UTC)[reply]
Indeed, radiation is a risk for those reasonably close to targets shortly after they are bombed. They may be a statistical increase in cancer risk for others, but that's about it. --Tango (talk) 19:38, 12 March 2010 (UTC)[reply]
If there is appreciable fallout in their food supply/land/water (as one would expect from a 1980s exchange of thermonuclears), the cancer and birth defect risks would be more than "statistical". --Mr.98 (talk) 01:56, 13 March 2010 (UTC)[reply]
That's true - as the quote says, it will be worst in North America and Eurasia. We can expect the southern hemisphere to fare better, but we're still talking a several degree temperature drop for years, which would be enough to ruin agriculture. Most of the population is in the north anyway. --Tango (talk) 19:38, 12 March 2010 (UTC)[reply]
I do not have the reference available, but I read a comment by someone who once was tasked with aiming US nukes at Russia, to the effect that it is easy to select the first several hundred targets, but by the time you are down to the 20,000th most important target, it is likely worth less than the bomb and its delivery system. 300 bombs could hit every US city with 100,000 people. 500 more could hit every military base in the U.S. Picture the Russian planner selecting target number 40,000: The courthouse in a small U.S. county? A 2 lane highway bridge? An electrical substation? A shopping center? Edison (talk) 18:03, 12 March 2010 (UTC)[reply]
A survey of basically all fiction about nuclear war, with some notes on its accuracy or lack thereof, by Paul Brians is available at [21]. Edison (talk) 18:21, 12 March 2010 (UTC)[reply]
Russia was known to have developed a crude anti nuclear missile technology (that the US may have duplicated too), which consisted of simply planning to detonate nuclear warheads in the direct path of incoming nuclear missiles. You don't have to be very precise that way. As I recall, the response to this technology was to plan to supersaturate certain targets, e.g. aiming 30 missiles at Moscow, so that at least one would get through and destroy the city. When you have a huge excess of warheads it is easier to make plans like that. Dragons flight (talk) 21:16, 12 March 2010 (UTC)[reply]
The Russian technology in question was the ABM-1 Galosh. It would have only been stationed around a few major cities. The American response was to ramp up MIRV development—make it so that there are 10 offensive missiles to every one defensive one. I don't think the US ever developed anti-missile technology of this sort, but they did have anti-bomber technology that worked this way (MIM-14 Nike-Hercules). Fun stuff. --Mr.98 (talk) 01:56, 13 March 2010 (UTC)[reply]
The non-negligible chances of missiles not launching cleanly and warheads not detonating also make it wise to send several to each target. Edison still has a point, though - even if you launch 30 at each target you still need about 1000 targets, which is more than enough to include every major city and military base. --Tango (talk) 21:32, 12 March 2010 (UTC)[reply]
Though it does get kind of ridiculous. In the 1960s (according to Richard Rhodes), Robert McNamara asked the air force how many missiles their force requirement calculations would say were necessary for Hiroshima. The response was that according to their up-to-date calculations, it would take three bombs of 80 kt each to shut down Hiroshima during World War II... or some 18X what was actually required to do "the job." The Eden book I cited earlier discusses in some detail the origins of these calculations and why they tended towards crazy levels of overkill. It is hard not to see some of this as just being about "we've got the money and can do it so let's do it" rather than serious questions about the necessity or consequences. Compare this with China's strategy, which is just about seeing how few missiles you can have and still maintain a viable second-strike deterrence—which they've concluded is about 200 or so. Much cheaper, less dangerous, and makes no pretensions of being a first strike force. (Which, for all that we talk about second-strike deterrence today, was in fact what the US policymakers were trying to do at the time, as the historical records have shown. Which is scary.) --Mr.98 (talk) 01:56, 13 March 2010 (UTC)[reply]
Speaking of games, this may be a subject relavent to game theory, zero-sum games and the Nash equilibrium. Shortly prior to the peak in Soviet warheads, the incident involving Korean Air Lines Flight 007 occured, and the Soviet Union may have planned a preemptive attack against the US in 1984. ~AH1(TCU) 02:51, 13 March 2010 (UTC)[reply]
The US certainly planned for preemptive (first strike) attacks against the USSR all through the Cold War. There were a load of close calls: Cuban Missile Crisis, Stanislav Petrov, Able Archer 83, etc. The idea that things were perfectly rational, stable, and that military and political officials in the US and USSR actually took deterrence seriously is demonstrably historically false! It is the kind of pat reassurance given by TIME magazine, but it is not serious history. --Mr.98 (talk) 19:48, 13 March 2010 (UTC)[reply]
Just read the Stanislav Petrov article. So the scenario described in the song '99 Red/Luftballoons' nearly actually happened? Holy crap. --95.148.105.19 (talk) 22:34, 13 March 2010 (UTC)[reply]
For another list of "close calls", take a look at World War III. ~AH1(TCU) 23:41, 13 March 2010 (UTC)[reply]

csi

whats a good article on here about evidence collection like dna fingerprints ect. in real life —Preceding unsigned comment added by 67.246.254.35 (talk) 05:42, 12 March 2010 (UTC)[reply]

Forensic science would be a good launching point; be aware that CSI is 99% bullshit. Its good theatre, but not good law work. Forensic science is a pretty good omnibus article on the topic. Much of CSI centers around the analysis of Trace evidence, but they also discuss many other aspects of forensics. Also see the CSI effect, a real problem in the forensic science world where people tend to believe that the stuff they see on CSI is somehow real, and it makes it hard for real forensic scientists to work with people like prosecutors and juries who have an unreal expecatation about what their job is like. (Full disclosure: My wife works in forensic science for a large police agency). --Jayron32 06:04, 12 March 2010 (UTC)[reply]

is it true they can take DNA from skin cells if someones arm or something brushes against an object? —Preceding unsigned comment added by 67.246.254.35 (talk) 06:33, 12 March 2010 (UTC)[reply]

Yes, but this also means that skin cells from many people are likely to be on any object. This makes it not so useful. You can say "Your skin cells are on the murder weapon", but they can say "So what, a dozen people's skin cells are on the murder weapon". StuRat (talk) 12:43, 12 March 2010 (UTC)[reply]
For lovers of CSI, I recommend: http://xkcd.com/683/ SteveBaker (talk) 13:31, 12 March 2010 (UTC)[reply]
This is also good. http://www.phdcomics.com/comics/archive.php?comicid=1156. --Mark PEA (talk) 16:21, 12 March 2010 (UTC)[reply]

orbit of moon

Is the expansion of the moon's orbit related to the expansion of the Universe? 71.100.11.118 (talk) 12:23, 12 March 2010 (UTC)[reply]

No, it's because the Moon is outside the Earth's geosynchronous orbit distance of 26,200 miles. Orbits below that distance will decay inward and those above that distance will decay outward, due to tidal interactions. StuRat (talk) 12:36, 12 March 2010 (UTC)[reply]
Does this explanation apply to the solar orbits of the planets? 71.100.11.118 (talk) 12:50, 12 March 2010 (UTC)[reply]
Yes, with all planets being outside that range, so having their orbits decay outward. However, the effect is tiny for the Moon, and even more so for the planets, as the tidal effects of the Moon on the Earth are far greater than the tidal effects of the planets on the Sun. Mercury, being the closest, would have the greatest effect, but even that might be too small to ever measure. StuRat (talk) 15:45, 12 March 2010 (UTC)[reply]
Also does this explanation mean that the moon has never orbited closer than 26,200 miles? 71.100.11.118 (talk) 12:54, 12 March 2010 (UTC)[reply]
Yes, unless something like a third body pushed it out of a closer orbit. StuRat (talk) 15:45, 12 March 2010 (UTC)[reply]
No, the rotation period of the Earth was originally ~8 hours / day, meaning geosynchronous orbit would have been much closer. Hence the moon also could have been closer (e.g. as close as 13000 miles). Most of the change in rotation rate is in fact attributed to the moon. Dragons flight (talk) 17:33, 12 March 2010 (UTC)[reply]
Good point. StuRat (talk) 19:10, 12 March 2010 (UTC)[reply]
The second paragraph at Orbit_of_the_Moon#Tidal_evolution_of_the_lunar_orbit is a very nice explanation of how the tidal bulges on the Earth coupled with the Earth's rotation cause the moon's orbit to increase. Does that help? Zain Ebrahim (talk) 13:03, 12 March 2010 (UTC)[reply]
So the interaction of Earth's spin, tides, are entirely responsible for expansion of the moon's orbit rather than the expansion of the Universe. 71.100.11.118 (talk) 13:20, 12 March 2010 (UTC)[reply]
Yes, Expansion of the universe has zero effect on the orbit of the moon. Dauto (talk) 13:28, 12 March 2010 (UTC)[reply]
Interestingly, by calculating how much the Moon's distance from Earth should increase if Hubble's law would be correct for such small distances, I get 28 mm/year, on the same order of magnitude as the 38 mm/year in measured increase of the distance, but I guess that is coincidence. Icek (talk) 18:05, 12 March 2010 (UTC)[reply]
I just did that calculation too (having not refreshed the page since you posted) and concur. It is a pretty amazing coincidence, but it is a coincidence. Our understanding of tides (which are based on very simple physics) predicts a recession of very close to the measured amount, so that understanding would have to be totally wrong for it to be caused by anything else. If our understanding of tides is wrong then our understanding of cosmology doesn't stand a chance of being right. --Tango (talk) 18:30, 12 March 2010 (UTC)[reply]

is there intelligent sleep like beauty sleep?

If you google beauty sleep, you get a whole lot of hits, at least some of them quite credible. Moreover, if you've ever seen the harried look of someone who's gone with very little sleep over many days, it is obvious that they're not quite well. It's ugly. Now, I wonder if there is an analogous effect on intelligence. Is someone intellectually "harried" by prolonged insufficient sleep? Do they absorb things they learn less, and so forth? I have a more specific question: when we say "sleep deprivation" we mean considerably less than 6 hours of sleep per day, maybe as little as 2-3 hours per night. (By the way, if anyone wants to edit the sleep deprivation article, it certainly doesn't define the amount of time of chronic sleep deprivation. Is it less than 3 hours per night, less than 4, than, 5, than, 6, than, 7, or what?)

Anyway I have a specific question. When people talk of BEAUTY SLEEP they don't mean "don't be sleep deprived", ie don't sleep just 3 hours. What they really mean is: don't sleep too little, only 5-6 hours. Sleep 8 or 9 hours instead.

Now, insofar as this works at giving people a healthier look, I wonder: does it have an intellectual effect as well? Does it make sense likewise to talk of 'intelligence' sleep like 'beauty sleep'.

Specifically, I know that it is hard to absorb and digest information if you are not able to sleep on it, severe, chronic sleep deprivation of only 2-3 hours of sleep per night wreaks havoc on one's cognitive capabilities...

but is there an EXTRA intellectual benefit, like the beauty benefit, to sleeping, say, 9 hours per night while learning a lot, as compared with only sleeping 6? Where is the cutoff on the return? Surely 12 hours per night doesn't make you any better at absorbing information and so forth than 9 hours per night, but surely 6 hours is much better for you with respect to the same cognitive effects versus 2 hours... so, is there a graph someone could show, or the specific cutoff points on the return?

I can imagine several possible graphs: I wonder which one is correct. Thank you. —Preceding unsigned comment added by 82.113.106.93 (talk) 13:09, 12 March 2010 (UTC)[reply]

I hope someone turns up with some studies to cite soon, but until then I can tell you from experience that there is such a thing as being intellectually not-at-full-strength because you don't get quite enough sleep for days on end: talk to some parents. I can also tell you that you're unlikely to find graphs in the form you want that can be generalised, because different individuals have different sleep needs, and this can vary depending on circumstance. For example: when I start a new job, I find I need more sleep for the first week or two. 86.178.167.166 (talk) 01:25, 13 March 2010 (UTC)[reply]
Not sure about specific studies, but some relavent articles include Sleep#timing, sleep and creativity, lucid dreaming, delayed sleep phase disorder, insomnia, psychomotor learning and health and intelligence. ~AH1(TCU) 02:33, 13 March 2010 (UTC)[reply]

Lorentz invariant

If two inertial systems (x, y, z, t) and (x', y', z', t') are moving relative to each other, with their origins coinciding at t = t' = 0, and at that moment a light pulse is emitted from the origin, then x2 + y2 + z2 - c2t2 = x'2 + y'2 + z'2 - c2t'2 = 0. To show that x2 + y2 + z2 - c2t2 = x'2 + y'2 + z'2 - c2t'2 is true in general, my textbook just applied the Lorentz transformations to (x, y, z, t) and crunched the algebra. Is there a simpler way (that is, by considering some sort of thought experiment or something) to show that x2 + y2 + z2 - c2t2 is invariant under a change of reference frame? —Preceding unsigned comment added by 173.179.59.66 (talk) 14:40, 12 March 2010 (UTC)[reply]

The book did that calculation just to double check the consistency of the theory. In fact that equation is nothing more than the statement that the speed of light is the same for all observers. This is the starting point of the theory and is usually taken as a postulate. No gedanken experiment is necessary. Dauto (talk) 17:04, 12 March 2010 (UTC)[reply]
After edit conflict:
Sure. Imagine the event E with coordinates (x,y,z,t) and (x',y',z',t'). Let's call the common origin event O. Let's work with c=1 and let's forget about y and z. I assume that's what the textbook does as well. (which book do you have?)
First assume t^2 > x^2. Assume a clock present at events O and E, which was also set to t"=0 at O. The unprimed/primed systems have a speed v resp. v' w.r.t. this clock. Then clearly v^2=x^2/t^2 and v'^2=x'^2/t'^2. Through the time dilation equations, the proper time t" on that clock satisfies t = t" / sqrt(1-v^2) and t' = t" / sqrt(1-v'^2). This implies that t" = t sqrt(1-v^2) = t' sqrt(1-v'^2). Substituting the values for v^2 and v'^2, we get the result.
Then assume t^2 < x^2. Assume a rod stretched between events O and E, and a system in which this rod is at rest. The unprimed/primed systems have a speed v resp. v' w.r.t. this system. (these v and v' differ form the previous ones!) Then clearly (see spacetime diagram!) 1/v^2=1-x^2/t^2 and 1/v'^2=1-x'^2/t'^2. Through the length contraction equations, the proper lenght x" of that rod satisfies x" = x / sqrt(1-v^2) and x" = x' / sqrt(1-v'^2). This implies that x / sqrt(1-v^2) = x' / sqrt(1-v'^2). Substituting the values for v^2 and v'^2, we get the result again. DVdm (talk) 17:45, 12 March 2010 (UTC)[reply]
Thanks, and I have Intro to Mechanics (Kleppner/Kolenkow). Oh and to Dauto: I don't think the Lorentz invariance thing is a statement that the speed of light is constant with respect to different inertial reference frames. The (x, y, z) coordinates don't have to be the point occupied by a light particle, just any point in general, so I don't think that you can extrapolate that because x2 + y2 + z2 - c2t2 = x'2 + y'2 + z'2 - c2t'2 = 0 for a beam of light, x2 + y2 + z2 - c2t2 = x'2 + y'2 + z'2 - c2t'2 is true for any event (x, y, z, t). If I'm wrong, please let me know! 173.179.59.66 (talk) 17:57, 12 March 2010 (UTC)[reply]
That's the reason why I elaborated. By the way, this Kleppner/Kolenkow is a very good textbook. Stick with it :-) - DVdm (talk) 18:43, 12 March 2010 (UTC)[reply]
You are correct, but what I said is also correct. There is no need for a gedanken experiment showing that the relativistic interval is an invariant. This is a postulate of the theory. In other words, you can define the Lorentz transformations as the set of transformations that respect the invariance of the relativistic interval the same way that you can define rotations as the set of transformations that respect the invariance of space-vector lengths. Dauto (talk) 18:42, 12 March 2010 (UTC)[reply]
Oh I see, thanks. 173.179.59.66 (talk) 02:23, 13 March 2010 (UTC)[reply]
I agree, but in many (older) books the invariance is not postulated, but rather derived from the "classic" postulates. DVdm (talk) 18:49, 12 March 2010 (UTC)[reply]

do you retain information better if you sleep on it?

this is related to the question above. My question is: do you retain information better if you've slept on it versus being tested the same day?

I don't mean because of the additional time since you learned it: say you compare learninng at 8 AM and being tested same day at 8 PM versus learning at 8 PM and being tested the next morning at 8 AM. Thank you. 82.113.121.104 (talk) 17:46, 12 March 2010 (UTC)[reply]

Several recent studies indicate this is the case. Even an afternoon nap can help. Graeme Bartlett (talk) 17:54, 12 March 2010 (UTC)[reply]
How did they control for tiredness? --Trovatore (talk) 17:55, 12 March 2010 (UTC)[reply]

According to Tony Buzan, your recall improves shortly after you have studied something (because your brain has sorted the information out) and then rapidly falls off. He recommender (and I found this works well) is to review the learn material 10 minutes after studying it, then review it again 24 hours later. The again after a week and once more after a month. This helps the information to be retained by the ‘long term’ memory. So, back to your question. You are likely to get higher marks if tested the same day. But it is better to keep reviewing it if you what to remember it long term. From Page 54, 55 ,56. Use your Head by Tony Buzan.--Aspro (talk) 18:21, 12 March 2010 (UTC)[reply]

I'm wondering too. I would like to test if this is the case for me. Guys here is my followup question:

can someone suggest a test methodology to control for everything but having "slept on it"

I would like to test if the above is true for me, and although the sample size will be 1, nevertheless that 1 subject is the one of greatest relevance to me :).

So can someone suggest an appropriate test methodology for me, for the simple task of trying to memorize and retain 10 of the multiplications from the 1000 x 10000 multiplication table, for example,

328x340=111520
274x781=213994
163x310=50530
494x65=32110
491x753=369723
969x954=924426
667x324=216108
77x813=62601
622x647=402434
196x249=48804

How should I do this in a way that will control TIREDNESS, control the stress or interference of other aspects of my life (e.g. I might concentrate better at night, since I wouldn't be doing something better anyway, but in the morning, I might be stressing out over something more important that I should be doing instead, and not concentrate fully and so forth). Any ideas? Thanks. 82.113.121.104 (talk) 18:09, 12 March 2010 (UTC)[reply]

You can't possibly do a well-controlled experiment with yourself as subject. The most common experimental paradigm is to have two groups of subjects, let's call them A and B. Group A learn the task, take a nap immediately afterward, sleep as much as they want to during the night, and are tested on the following day. Group B do the same things except without the nap. Thus both groups have the same learning experience and are well-rested during testing -- the only difference is the nap. For a recent review of the literature pertaining to this topic, you might look at PMID 19251443, if you have access to it. Looie496 (talk) 19:03, 12 March 2010 (UTC)[reply]
Obviously I can't do a double-blind experiment with myself as a subject; that's a given. But why couldn't I do a well-controlled experiment with myself as a subject, granted it might take longer, but you can see from my methodology below that you can still have a control...
Now my deeper problem with the methodology you mention is that BOTH groups a and b get to "sleep on it"!!! I am interested in the difference between being able to sleep on it, versus not having slept between learning the material and being teested.
To this end, what about the following test methodology:

For the next 20 days, every morning I get up at 6, every morning at 8 I put aside 15 minutes for the Methodology, every night I put aside 15 minutes at 8 PM for the Methodology and go to sleep at 10 PM.

  • I group my these 20 days into groups of two (call them day1 and day2 within each group). At the onset, I randomize each pair of days into either being a "group a" pair of days or a "group b" pair of days.
  • Then, each day goes as follows:
  • If it is day1 of a group a, I do the studying in the morning slot, and take my test in the evening slot.
  • If it is day2 of a group a, then I don't do anything in either the morning or evening slot.
  • If it is day1 of a group b, I don't do anything in the morning slot. In the evening slot I do the studying.
  • If it is day2 of a group b, I take the test in the morning slot. I don't do anything in the evening slot.


Now what could possibly be wrong with that methodology? It has both a control (group a pairs of days and group b pairs of days) and beyond this, it has an equivalent 12 hours between the studying and the being tested - the only difference is one of the groups includes 8 hours of sleep in that 12 hours, and the other group does not. Now, if there is a statistically significant difference in this well-controlled study on myself, it can be argued that the difference is because I am "more tired" at 8 PM than 8 AM, or alternatively phrased, "more alert" at 8 in the monring, after having awoken at 6 AM. Also the effects of any coffee I would drink would have to be normalized (ie I would not drink coffee until after hte test in the mornning). Same for taking my showers. Is there any other way in which the methodology I've just proposed would fall short? Thank you. 82.113.121.104 (talk) 19:46, 12 March 2010 (UTC)[reply]

The problem is that test performance is influenced by motivational factors, and it's impossible for you to ensure that you're equally motivated to perform well under all conditions. Even if you don't realize it, you're sure to have some motivation to want one outcome rather than another. Looie496 (talk) 20:07, 12 March 2010 (UTC)[reply]
so what do I have to do? Do the test with my brother, but not tell him what I'm testing, ie he doesn't know if it's significant whether the recall test is the next day or the same day. Instead, he just thinks it's a multiday test? I guess here you're going to tell me that that's good, but not good enough: it is single-blind. Instead, I need to get my brother to do the test with someone else, WITHOUT letting my brother know that what is really being tested is whether the subject has slept on it. Obviously all this machination just to get the result "no difference" is ridiculous, when I might get the same result myself anyway, so you guys must really be convinced there's a big difference... :) 82.113.121.104 (talk) 22:51, 12 March 2010 (UTC)[reply]
If you want a real study, you need more than one subject. You need, at a minimum, a control group and a test group. By just testing your brother, you are getting nothing more than an anecdote - which is not data. Further, your statement appears to suggest that you think everyone here is involved in a conspiracy to keep a secret from you. That is silly. -- kainaw 22:56, 12 March 2010 (UTC)[reply]
Well, you'd need multiple subjects if you were doing this for publication, or even for a school project. If you're just trying to get an answer for yourself, you could do it by repeatedly testing a single subject -- but even then it's important that the subject not be aware of what you are looking for, in order to avoid having your subject subconsciously sway toward the result you want (or against it!). Double-blind experiments are always nice, but generally not possible in sleep studies -- you can't blind somebody to the difference between being awake and being asleep. Looie496 (talk) 23:21, 12 March 2010 (UTC)[reply]
Referring to your test numbers, some numbers have special properties including 142857. Another subject related to functioning versus sleep is sleep and creativity. ~AH1(TCU) 02:11, 13 March 2010 (UTC)[reply]
There is also the question of what you mean by "retain" information, obviously you could take this as being able to repeat a set of numbers. But for something like understanding how a set of variables interact, sleeping can help you process the data, hence why you sleep on a decision. Now in this case you might "retain" the meaning of the information better but not be able to write down the table, equation etc precisely. 82.132.139.214 (talk) 02:20, 13 March 2010 (UTC)[reply]

center of mass

Technically, isn't it the Earth/Moon center of mass that orbits the sun making the Earth closer or further away from the Sun at times during the rotation about the center of mass? 71.100.11.118 (talk) 18:24, 12 March 2010 (UTC)[reply]

Yes, but the Earth is so much larger than the Moon that the centre of mass of the two is inside the Earth. That means the Earth wobbles a bit, but nothing more. --Tango (talk) 18:40, 12 March 2010 (UTC)[reply]
And this wobble (a few thousand km ?) is insignificant compared to the distance caused by the Earth's slightly elliptical orbit, which is on the order of 5 million km. StuRat (talk) 18:58, 12 March 2010 (UTC)[reply]


Deriving the Lorentz transformations

It seems that the common approach to obtaining the equations for the Lorentz transformations are by guessing at their form and then, by considering four seperate situations, determining the constants. From these equations, things like time dilation and length contraction can be worked out. Now, my goal was to go the other way around: starting from time dilation and length contraction, arrive at the Lorentz equations.

Suppose that in the reference frame O, the reference frame O' is moving at a speed v in the x-direction, with their origins coinciding at t = 0. An event E occurs at (x, y, z, t) in the O frame. It's straightforward to show that y' = y and z' = z. Next I considered x'. In the O frame, the distance between O' and E is x - vt. Because the ruler O' uses is shortened by a factor of γ , she will then measure the distance x - vt as being greater and O measures it, by a factor of γ. Thus, x' = γ(x - vt).

However, I'm having trouble with t'. I know that t' = γ(t - vx/c2). I assume that the -vx/c2 term comes from that, because O' believes that she's at rest, when the light emitted from the event reaches her, she doesn't treat herself as moving into the light, and thus there's a discrepancy as to how long before the light reaches O' did the event actually happen. Unfortunately, I can't arrive algebraically at this term. Finally, the gamma factor. I assume this comes from time dilation, but the wouldn't O' 's clock be running slower? So wouldn't the term have to be 1/γ? Can someone please tell me how to get the final Lorentz term this way? I know there are probably other, easier routes, but for personal reasons I would like to know how to do it this way. Thanks a lot! 173.179.59.66 (talk) 18:31, 12 March 2010 (UTC)[reply]

It seems that you're failing to take into account that the two observers perceive different simultaneities.Dauto (talk) 18:50, 12 March 2010 (UTC)[reply]
You might be interested in Bondi k-calculus. As I recall your desired approach is pretty much exactly that. Although the article isn't exactly detailed. 129.234.53.144 (talk) 19:43, 12 March 2010 (UTC)[reply]
Hmmm not really, Bondi starts from the Doppler shift and works from there, instead of from time dilation (his equations have a bunch of k's). And as for the simultaneity comment, I thought that by considering how long it would take the light to reach the observer in each frame, simultaneity would be accounted for (as that is how differences in simultaneity arise). And then there's the issue with the gamma factor, which I think should be 1/γ. Sorry, I'm still lost! 173.179.59.66 (talk) 04:01, 13 March 2010 (UTC)[reply]
"...the distance between O' and E is x - vt." => That is a distance between a pair of simultaneous events in the unprimed system. So these events are not simultaneous in the primed system. Then you arrive at "Thus, x' = ?(x - vt)." But that is a coordinate of the event E in the primed system. As such it denotes the distance between a different pair of events. When you draw a spacetime diagram, you see what is going on. A piece of advice: always think in terms of events, and always draw a spacetime diagram. DVdm (talk) 11:57, 13 March 2010 (UTC)[reply]
So then, how would you fix this conundrum with t'? 173.179.59.66 (talk) 12:16, 13 March 2010 (UTC)[reply]
I don't think there's anything to fix, and we surely haven't fixed the trouble with x'. We merely established that not only you are "having trouble" with t', but with x' as well. DVdm (talk) 13:12, 13 March 2010 (UTC)[reply]


The point (0, 0, 0, t - vx/c2) is the point along the O frame path that's simultaneous to E as viewed from the O' frame. In the O' frame, clocks in the O frame run slower by a factor of γ. To the O' observer, when event E happens, the clock in the O frame reads t - vx/c2, and so the amount of time that has elapsed in the O' frame is actually γ(t - vx/c2). Rckrone (talk) 20:30, 13 March 2010 (UTC)[reply]
So basically, what you're saying is that we must look at the perspective of O', not O. Why would this be (afterall, we're starting with O 's coordinates, and the time dilation/length contraction all occur in O' 's frame, but this approach seems to yield the wrong answer). Sorry if I'm being a bit slow, I really want to understand this, subtleies included. 173.179.59.66 (talk) 21:16, 13 March 2010 (UTC)[reply]
We're trying to find the O' time coordinate of E, which is the amount of elapsed time that has been experienced by O' at the point where O' views event E as being simultaneous to her. So we need to consider the points simultaneous to E in the O' frame and how much time has passed in the O' frame up to those points. We already know what E looks like in the O frame: it's at point (x, y, z, t). Rckrone (talk) 21:32, 13 March 2010 (UTC)[reply]
Exactly. That's why I mentioned earlier that 173.179.59.66 was failing to take the different simultaneities into account. There is a way to avoid having to deal with simutaneities and time dilations. I'll post it later because I'm busy right now. Dauto (talk) 22:43, 13 March 2010 (UTC)[reply]
Things are making much more sense now, I eagerly await your posts! PS: how did you get the vx/c2 term?173.179.59.66 (talk) 01:18, 14 March 2010 (UTC)[reply]

Okay, I've made some headway myself, and have managed to understand why it should be t' = γ(t - ...) instead of t' = (1/γ)(t - ...). However, the vx/c2 term still eludes me. Here's the work I've done: I looked at the O' frame, with the goal of getting (x,y,z,t) in terms of (x',y',z',t'), which can then be easily switched to get (x',y',z',t') in terms of (x,y,z,t). I should get t = γ(t' + vx'/c2). Now, the gamma factor comes from clocks running slower in the O frame relative to the O' frame. The vx'/c2 should come from difference in percieved signal delay between reference frames. In the O' frame, the light from event E reaches O at t' + x'/(c-v). From O 's perspective, the signal delay should only be x/c, so she will conclude that E happened at t' + x'/(c-v) - x/c. x' = x/γ, so this becomes t' + x'(1/(c-v) - √1-v2/c2/c). Problem: this doesn't seem to algebraically reduce to t' + vx/c2. Where's my mistake? 173.179.59.66 (talk) 03:03, 14 March 2010 (UTC)[reply]


To arrive at the expression for we have to take two things into account - A space dilation/contraction and an offset. We can for instance assume that there is a ruler of length at rest with respect to the primed coordinate system so that one end of the ruler coincides with the coordinate origin at all times and the other end coincides with the event that happens at . Now, that ruler is at rest at the coordinate system so is its proper length. The length of the ruler on the unprimed coordinate system gets space contracted, . That was the space dilation/contraction. The offset comes about because ot the relative motion between the two coordinate systems. From the point of view of , by the time at which the event happens the origins of the two systems will be already a distance apart and that distance - the offset - must be added to the length of the ruler to get the coordinate , giving us the equation . That last equation can be solved for and we find .


It so turns out that in order to arrive at the expression for we also must take into account a time dilation/contraction and an offset. We can place a watch at rest with respect to system at the coordinate and start the watch exactly at the instant the two origins coincide (from the point of view of ). The watch will mark the time when the event happens. Now, that watch is at rest at the coordinate system so is a proper time, and this time interval will be seen time dilated from the point of view of the system . That is . That was the time dilation/contraction. The offset comes about because from the point of view of the system the watch was not started simultaneously with the time the two origins coincided. There is a delay given by . must be subtracted from to get the coordinate . We get then the equation


Even though the space offset due to relative motion is very intuitive, the time delay due to non-simultaneity isn't. It is to our advatage if we manage to find the transformation without ever having to mention simultaneity or time dilation. To do that we start from the last equation of the first paragraph found using space contraction and relative motion only
We also have a similar equation obtained for in terms of and . It's the same equation but with a sign difference because from the point of view of , is moving in the oposite direction
Substituting from the first equation into the second,
Voilla. Dauto (talk) 03:38, 14 March 2010 (UTC)[reply]
Wow that was exactly what I was looking for, thanks a lot! Really, that was a great help. I just have one final question though (sorry!): what was wrong with the signal delay approach I used above? I'm guessing it's another issue with simultaneity, but I just can't see were the problem is (or how it can be resolved, if I was inclined to use the signal delay way to avoid having to imagine clocks placed along the x-axis). I know it may seem trivial, but I want to have a thorough understanding of special relativity, and if I can't see why a certain approach is flawed then I can't say I really do. 173.179.59.66 (talk) 04:18, 14 March 2010 (UTC)[reply]
Never mind, I figured it out: I was multiplying my gammas at the wrong time. Much thanks again! 173.179.59.66 (talk) 05:35, 14 March 2010 (UTC)[reply]

tidal forces

For two absolutely solid bodies (such as pure diamond crystal matrix) orbiting each other and assuming no tidal forces then the spin of either body results in no geosynchronous orbit such that the orbit neither decay inward or outward regardless of the orbital distance, is that correct? 71.100.11.118 (talk) 18:33, 12 March 2010 (UTC)[reply]

You can still get friction and gravitational waves, which can remove angular momentum away from your system. Graeme Bartlett (talk) 18:48, 12 March 2010 (UTC)[reply]
The assumption that diamonds don't undergo tides is incorrect. Any matter will be affected by gravitational forces, and thus undergo tides, although solids deform much less than fluids. StuRat (talk) 18:53, 12 March 2010 (UTC)[reply]
There will always be tidal forces. I suppose if the body is sufficiently rigid that those forces don't result in significant bulges then we can discount them, though. In that case, there won't be any tidal interactions resulting in decay, in any direction for any orbit. There are other factors that will change the radius of the orbit, though - atmospheric drag, for instance (particularly for low orbits), and interactions with other objects. --Tango (talk) 18:57, 12 March 2010 (UTC)[reply]

Animals that feed only on animals of their own kind

Which animals eat nothing but other animals of its own kind? I know there are lots of animals that practice cannibalism but all cannibal species I think of usually eat other stuff too and I can't even think of any cannibal species that feeds primarily (let alone exclusively) on the animals of its own kind. I am aware that such behaviour would be very inconvenient for the survival of the species but some animals routinely eat their young and still thrive. Surtsicna (talk) 19:47, 12 March 2010 (UTC)[reply]

It's pretty obviously impossible -- since digestion is less than 100% efficient, the biomass of such a species would have to steadily decrease. Looie496 (talk) 19:59, 12 March 2010 (UTC)[reply]
Exactly. There would be no way for energy to enter the species. Even with 100% efficient digestion, not all that energy is converted to biomass. Much of it is used for locomotion, for example. --Tango (talk) 20:05, 12 March 2010 (UTC)[reply]
Agreed. You could, however, have a small portion of a species which cannibalizes the rest. I would expect that this portion would then evolve into a new species. StuRat (talk) 20:07, 12 March 2010 (UTC)[reply]
Whether they evolve into a new species would depend on whether they breed with the rest of the population or not. If they do, there won't be any speciation. It is common for, for example, dolphin pods to have different diets even in the same environment, but they still breed more generally. --Tango (talk) 20:42, 12 March 2010 (UTC)[reply]
I wouldn't expect much interbreeding between hunters and prey, both because the prey will run and hide and because the hunters will kill the prey if they find them. StuRat (talk) 16:00, 13 March 2010 (UTC)[reply]
Hmm. That argument implies all biomass comes from ingestion. What about if the animal were symbotic with something photosynthesising which lived in them? Not that I can think of any such but I am not clear it is technically impossible for an animal with a second such source of biomass to ingest only others of the species. --BozMo talk 21:14, 12 March 2010 (UTC)[reply]
In that case, it would have to either derive most of its energy from the other source or reproduce faster than it can eat. The first satisfies OP's question in letter but not in spirit - it would not technically be "eating" if it were to derive energy through photosynthesis. The second is unlikely given the normal ratio of consumption of nutrients to species reproduction found in other animals. That is to say, in my experience most creatures eat more often than they reproduce. Coreycubed (talk) 21:31, 12 March 2010 (UTC)[reply]
Well biology is not my subject but there are animals as far as I can tell which eat nothing (eg a Giant tube worm which derives its energy from non digestive processes and has no digestive tract). So variants could conceivably exist which ate only its own kind. But you have to be a long way out of the ordinary, and certainly not close to your ordinary ratios. --BozMo talk 21:51, 12 March 2010 (UTC)[reply]
Yes, but as Coreycubed says, that only satisfies the letter of the OP's question, not the spirit. --Tango (talk) 00:05, 13 March 2010 (UTC)[reply]
I seem to recall a certain species of spider where the young eat their mother for nutrition, probably Segestria florentina. Spider cannibalism is also relavent but those species do not solely depend on cannibalism for foon. ~AH1(TCU) 02:06, 13 March 2010 (UTC)[reply]
Interesting, but I'm sure they eat more than just their mother during their lives. --Tango (talk) 02:16, 13 March 2010 (UTC)[reply]
Ants have three or more genders. If the workers, or whichever ones are non-reproductive, went around fattening themselves up and then the soldiers and queens ate them it could work like that, in the sense of enough energy gathered. ~ R.T.G 02:56, 13 March 2010 (UTC)[reply]
I wouldn't say they had more than two genders - workers are female, just infertile females. Also, I think soldiers are a type of worker. The males (drones) tend to do very little. All of that aside - what you describe would work, but it doesn't meet the OP's requirements since the workers are eating something other than their own species. --Tango (talk) 03:10, 13 March 2010 (UTC)[reply]

Terminal cancer

If someone is in the final stages of terminal cancer, why is nutrition denied?--79.76.188.14 (talk) 21:51, 12 March 2010 (UTC)[reply]

You need to be careful with terminology here, as nutrition and food aren't the same things. Nutrition is essential to survive, so to keep the patient comfortable they would need to be feel at least partially nourished, otherwise the individual will starve. As far as I know, in terminal cancer (i.e. where there will be no further medical intervention other to make the patient comfortable), food wouldn't be denied to the patient, but rather the patient may be unable to take food orally because of severe weakness associated with late stage cancer. Someone feel free to correct me there. Regards, --—Cyclonenim | Chat  23:04, 12 March 2010 (UTC)[reply]
Sorry I may have misunderstood your question, are you referring to nutrition being denied by the medical professionals, or nutrition to cells being reduced and why that occurs? Regards, --—Cyclonenim | Chat  23:22, 12 March 2010 (UTC)[reply]
Nutrition denied by medical professionals--79.76.188.14 (talk) 00:03, 13 March 2010 (UTC)[reply]
In that case, as Cyclonenim says, it simply isn't. Where have you got that from? Palliative care certainly involves feeding patients if they are willing and able to eat (and giving them intravenous nutrition if they can't/won't eat and it is appropriate to do so). --Tango (talk) 00:09, 13 March 2010 (UTC)[reply]
No, I think it is sometimes. It's a form of passive euthanasia. Generally we're talking about a patient in a coma or near-coma state, and who, if he did regain consciousness, would not likely enjoy it. --Trovatore (talk) 00:12, 13 March 2010 (UTC)[reply]
True, it is used for coma patients (although not usually in cases where they think there is a significant chance of the patient waking up). Do cancer patients typically end up in comas, though? Brain tumours sometimes cause comas, but I wouldn't expect other forms of cancer to. --Tango (talk) 00:55, 13 March 2010 (UTC)[reply]
When the body is unable to absorb nutrients due to organ failure but the nutrients continue to be supplied to the patient, conditions such as edema can occur. See also medical ethics. ~AH1(TCU) 02:01, 13 March 2010 (UTC)[reply]
True, but by that point the patient would be at most hours from death anyway, wouldn't they? Not feeding someone for a few hours isn't unusual, even if they are perfectly healthy. --Tango (talk) 02:13, 13 March 2010 (UTC)[reply]
Some cancer patients lose the ability to process oral food weeks or even months before they ultimately die. Fluids and some nutrition can be supplied intravenously for a time, but the lack of solid food and difficulty absorbing nutrition can contribute to the sense of wasting away that is not uncommon in end stage cancer patients. Dragons flight (talk) 19:56, 13 March 2010 (UTC)[reply]
Absolutely, but that isn't nutrition being withheld. --Tango (talk) 21:22, 13 March 2010 (UTC)[reply]
Nutrition is sometimes withdrawn from terminal patients, with the consent of the relatives, in certain jurisdictions, as a legal method of euthanasia. The logic is that the patient "dies on their own", versus being injected with a poison, which would be considered murder. Of course, if they are on a ventilator, then that can be turned off as a quicker method of euthanasia. StuRat (talk) 02:09, 13 March 2010 (UTC)[reply]
There have from time to time, including recently, been reports in the UK press and media of, and legal investigations into, cases where elderly patients (some terminally ill, others apparently not) have allegedly been wrongly or over-prescribed with opiate drugs or denied nutrition and water, leading to their quicker and/or unnecessarily unpleasant and/or unnecessary deaths. One case is referenced here; I recall others I don't feel inclined to dig for. Such cases, obviously legally and emotionally sensitive, may reflect possible incompetence or wrongdoing on the part of some medical staff. 87.81.230.195 (talk) 04:23, 13 March 2010 (UTC)[reply]
The issue of over-prescribing opiates is often one of double effect - painkillers are given in the quantity necessary to deal with the pain despite the doctor knowing that dose will kill the patient. --Tango (talk) 21:22, 13 March 2010 (UTC)[reply]
So, if nutrition was not denied intravenously (say), would the patient live longer?--79.76.188.14 (talk) 22:49, 13 March 2010 (UTC)[reply]
Obviously the main factor here is how severe the cancer is, but without nutrition I imagine the patient would die sooner as they would be weaker from lack of energy. No source for that, just seems common sense. Regards, --—Cyclonenim | Chat  23:03, 13 March 2010 (UTC)[reply]

lab rats

where are lab rats and mice bought from for research purposes? —Preceding unsigned comment added by Thekiller35789 (talkcontribs) 23:36, 12 March 2010 (UTC)[reply]

From companies that sell rats and mice for research purposes. E.g., [22] See laboratory rat. alteripse (talk) 23:38, 12 March 2010 (UTC)[reply]


i already read that it didnt help —Preceding unsigned comment added by Thekiller35789 (talkcontribs) 03:52, 13 March 2010 (UTC)[reply]

here are several commercial suppliers: [23] [24] [25]. Note that most animal houses (individual labs rarely keep their own rats and mice these days - instead they are housed at a central specialised facility) would only buy animals if they needed a new strain or an existing strain was lost due to disease etc. Otherwise it is far easier and cheaper to breed their existing mice.131.111.185.68 (talk) 08:45, 13 March 2010 (UTC)[reply]
Also, while I haven't done work with animals myself (well, other than fruit flies) I would imagine that labs quite often give more specialised strains as gifts to other labs which need them. This is quite common practice in (academic) science - it is much less hassle than trying to agree on a price and the sharing of resources means that everyone can get on with their work without spending six months duplicating what another lab has already done.131.111.185.68 (talk) 08:54, 13 March 2010 (UTC)[reply]

galactic spiral arms

Given enough time will the stars that fill the galactic plane accrete and form "stellertoids" (similar to planetoids only made up of stars that have accreted into super giant stars or black holes) and become spaced out into planetoid like orbits around the galactic center? 71.100.11.118 (talk) 23:52, 12 March 2010 (UTC)[reply]

Not really. Depending on what the ultimate fate of the universe is, they might well all end up in black holes, but that will be long after they stop being fusing stars (they'll be neutron stars, white/black dwarfs, smaller black holes, etc.). I suppose it is possible for stars to merge, but it would be a very unusual event (and I'm not quite sure what would happen - it would depend on the masses of the stars involved, certainly). I think you may be a little confused about what a planetoid is, too - a planetoid is a small planet-like body. In the early solar system, planetoids would have accreted into planets. Given your terminology, I think you may be thinking planets merge together to form planetoids, which isn't the case (I apologise if I'm the one misunderstanding). Also, the orbits of stars around the galactic centre are very similar to the orbits of planets around stars - the spiral arms are just a kind of pressure wave caused by complicated interactions, individual stars follow normal orbits (they are slightly different because the mass of the galaxy isn't dominated by the centre like that of a solar system is, but that isn't very important, it just affects the numbers a little). --Tango (talk) 00:17, 13 March 2010 (UTC)[reply]
No, actually I am thinking of planetoids as the precursors to the formation of planets. Try to decide what terminology to use when composing an imaginary situation always runs the risk of bringing with it a misconception or the wrong idea (my naughty speller doesn't help either!). Basically what I'm asking is that in absence of any other interference such as collisions or merges with other galaxies or the Universe ending prematurely from the Big Rip or Crunch if accretion of stars into planet like orbits around the Galactic center is a reasonable consequence. (The main reason for the question is in consideration of how much space can separate such orbits since the motion of the solar system planets form spirals when you connect them as dots for regular periods of time.) —Preceding unsigned comment added by 71.100.11.118 (talk) 07:22, 13 March 2010 (UTC)[reply]
Also, before your scenario or something similar could occur, the Milky Way could likely collide with the Andromeda Galaxy to form Milkomeda. This would disrupt the spiral structure, and our galaxy could turn more elliptical. The colliding pair could also absorb numerous other galaxies such as the Large Magellanic Cloud, Small Magellanic Cloud and the Triangulum Galaxy, further disrupting the "clumps" of stars and gas in the galaxies. ~AH1(TCU) 01:55, 13 March 2010 (UTC)[reply]
The merging of stars in the galactic disc is an excedingly rare event. Mergers are still rare but not completely negligible on higher density enviroments such as blobular clusters. Read blue stragglers to see what happens to stars after merging. Dauto (talk) 17:10, 13 March 2010 (UTC)[reply]
I assume Dauto has mistyped "blobular clusters" for Globular clusters, but I like this new term and can imagine someone applying it in the future, perhaps to non-symmetrical globulars :-). 87.81.230.195 (talk) 21:52, 13 March 2010 (UTC)[reply]
However, in the case of stars that collide with sufficient energy, they may explode in a hypernova, or if pulsars are involved then magnetic or gravitational waves may be released (I seem to recall an article involving a specific event where such a collision released energy that hit the earth's atmosphere but I can't find it right now). ~AH1(TCU) 23:33, 13 March 2010 (UTC)[reply]

March 13

Sodium bicarbonate

I noticed that if you burn a chlorate, you get the chloride (with oxygen gas), and if you burn a carbonate, you get the oxide (with carbon dioxide gas), so what does burning a bicarbonate, such as sodium bicarbonate, produce? --70.250.214.164 (talk) 00:25, 13 March 2010 (UTC)[reply]

I don't think you really mean "burn" — more like "heat" or perhaps "pyrolyze". If I recall correctly, heating sodium bicarbonate will first drive off water, producing sodium carbonate. This reduces the question to the previous case, which you have already solved. --Trovatore (talk) 00:34, 13 March 2010 (UTC)[reply]
You could try reading the article; it answers the question directly. Looie496 (talk) 00:37, 13 March 2010 (UTC)[reply]

The mass of a singularity

Don't give out to me now. A principle of the singularity, in the big bang anyway, is that the mass could be fit into an infinitely small space. To say "infinitely small", there is no requirement of the word small, right? There is zero size, right? If that follows, could there be a singularity of some sort that requires zero mass? Could they peel open a quark, for instance, and a singularity or something pops out? Is there interesting theories or anything like that? ~ R.T.G 02:45, 13 March 2010 (UTC)[reply]

Quarks do have mass, so I'm not sure what you mean. However, singularities are usually just interpreted as a sign that we've made a mistake rather than as a genuine physical object. When the maths results in a singularity it is basically saying "I don't know". When we get a good working theory of quantum gravity, hopefully the singularities in black holes and the big bang will disappear. --Tango (talk) 03:03, 13 March 2010 (UTC)[reply]
Often in sci-fi there is something about where they have cracked open an atom or interfered with the density of a star opening a rift which could destroy a universe. Couldn't that sort of potential be a basic element? I suppose trying to write into it I am thinking well maybe pigs will fly too but the theory about it is hardly going to be in aerodynamics. Maybe if a butterfly flaps a bit all these storms will stop for the summer. Oh well never mind me. Thanks for answer anyway. ~ R.T.G 04:21, 13 March 2010 (UTC)[reply]
Tango, this is the second time recently that you've stated (in so many words) "singularities are usually just interpreted as a sign that we've made a mistake rather than as a genuine physical object. When the maths results in a singularity it is basically saying "I don't know"." First, what is your source for this? Second, the overwhelming consensus is that, e.g., the singularity of a black hole has infinite density; literally dozens of sources for this statement could be found in one bookshelf of one library. If you personally believe singularities are nonsense, that's fine -- but it is not the overwhelming consensus ... which equals the "truth" for our purposes here. 63.17.60.8 (talk) 05:33, 13 March 2010 (UTC)[reply]
It's very well known that general relativity, which predicts singularities in black holes and for the Big Bang, breaks down at such extreme energies. This is what's behind the search for quantum gravity, and is not at all controversial.
Also, to the OP: white hole? --99.237.234.104 (talk) 07:13, 13 March 2010 (UTC)[reply]
Gravitational singularity, for reference. Indeterminate (talk) 09:50, 13 March 2010 (UTC)[reply]
Indeed - Tango is correct. Our gravitational singularity article says "Many theories in physics have mathematical singularities of one kind or another ... This is generally a sign for a missing piece in the theory, as in the Ultraviolet Catastrophe and in renormalization". In the case of gravitational singularities the canonical view, which I believe originated with Penrose and Hawking, is that the physcially unrealistic conditions predicted by general relativity can only be resolved by an as yet unknown theory of quantum gravity. Gandalf61 (talk) 10:04, 13 March 2010 (UTC)[reply]
If something had zero mass but finite space, then it would either be considered a vacuum or nothing, but even a pure vacuum contains energy in the form of dark energy and vacuum energy. However even a singularity may have "volume" in the sense that it contains a geometrical shape but possibly in a different dimension as in the case of a ring singularity. Also, you may be interested in micro black holes. ~AH1(TCU) 23:24, 13 March 2010 (UTC)[reply]

the frog still alive

how possible?


http://www.youtube.com/watch?v=QLz8RK4teHM&feature=related —Preceding unsigned comment added by Thekiller35789 (talkcontribs) 04:46, 13 March 2010 (UTC)[reply]

Twitching is not the same as living. See the article Galvanism which mentions the 18th century discovery that a severed frog's leg can be made to twitch. Cuddlyable3 (talk) 23:42, 13 March 2010 (UTC)[reply]
Frogs are pithed before dissection, severing the spinal column so they don't kick their legs. Nerve impulses from the spinal column can cause the legs of a dead/dieing frog to kick. All it takes is a volt or so. In the early 19th century, 1804, a hanged and dead man, Mr Foster, was used with his spine severed for electrical experiments. [26], [27], [28], [29] by Giovanni Aldini. A battery connected to muscle groups could make the dead man move his arms and legs, move his eyes around, and extend his arm. Frankenstein, anyone? Mary Shelley doubtless read of these experiments. A dead horse would kick his leg when a battery was applied, A chicken with her head chopped off can run around flapping her wings for a minute or so. Still dead. Edison (talk) 04:43, 14 March 2010 (UTC)[reply]

Was there sex in the biosphere?

? —Preceding unsigned comment added by 67.246.254.35 (talk) 05:30, 13 March 2010 (UTC)[reply]

If you're talking about this biosphere, then the answer is an unquestionable "Yes". If you're referring to Biosphere 2, then the answer is still likely "yes" as this source suggests. Dismas|(talk) 07:14, 13 March 2010 (UTC)[reply]
Or if you are asking about sex in space, see that article. ~AH1(TCU) 23:19, 13 March 2010 (UTC)[reply]

Cylindrical Fuels rods??any

I have made an characterisation and optimisation study on flames with open top enclosures<Cylindrical Radial enclosure only>.It is already selected for an international Conference.I have problem in finding s field of application for it.Please anybody suggest.

Thanks in advance —Preceding unsigned comment added by SCI-hunter (talkcontribs) 05:41, 13 March 2010 (UTC)[reply]

If you can describe these fuel rods better, we may be able to help. Is this just a cylinder of flammable material ? StuRat (talk) 13:42, 13 March 2010 (UTC)[reply]
If it it can achieve a clean burn, then you have my vote to use it as a neat and nifty heat source for my one gallon Bagna càuda and save filling the room with the oily smell of burning paraffin wax. --Aspro (talk) 14:11, 13 March 2010 (UTC)[reply]

Yes i have modelled the situation with a candle flame.hence it is a cylinder of flammable material.Any way what is banga cauda stated by Aspro.Some more application preferably in industries would be welcomed. Thanks SCI-hunter (talk) —Preceding undated comment added 15:20, 13 March 2010 (UTC).[reply]

You are the same guy who thought a common candle was somehow related to a "CANDLE nuclear reactor" - which I think we can conclusively say it isn't. So somehow this confusion has resulted in you getting this "study" into an international conference!?! OMG! That's too funny! I strongly recommend that you call up the conference panel, apologize profusely and withdraw your paper ASAP. SteveBaker (talk) 15:40, 13 March 2010 (UTC)[reply]
Steve, I don't think English is their first language, so mistakes like that don't mean they are an idiot, it just means they don't speak English well. StuRat (talk) 15:54, 13 March 2010 (UTC)[reply]
OK, assuming you are talking about something with the characteristics of a candle, namely burning at one end by combustion with environmental oxygen, let's see what applications we can come up with. I would assume that this would be more expensive than fuel sources such as propane or natural gas. The advantage may then be that it's more portable, and stores well. If so, uses where other fuel supplies aren't available come to mind, such as camping or for emergencies (in a car road kit, for example). Perhaps this could be used to start a camp fire in damp weather ? I wouldn't think there would be much industrial use, as industry is likely to have cheaper sources of energy avaialble. StuRat (talk) 15:54, 13 March 2010 (UTC)[reply]
How about using as a heat source for a Fondu (that is also is like a Bagna càuda but more popular with trailer trash and nouveau riche )… A better fondu heat source could be the biggest advance in human dinner party experience since Boy Scouts first sat around the camp fire together and roasted their nuts.--Aspro (talk) 16:44, 13 March 2010 (UTC)[reply]

MATERIALS USED IN BATHROOMS

I WANT THE FULL DETAILSSSSSSS —Preceding unsigned comment added by Waqasiqbal175615 (talkcontribs) 08:05, 13 March 2010 (UTC)[reply]

Tile, porcelain, metal, wood, drywall, paint, glass. Do you have a more specific question? Comet Tuttle (talk) 08:26, 13 March 2010 (UTC)[reply]

is it possible to boil water through the power of the mind alone?

If I put a pot of room temperature water on the stove, which is cold and turned off and to remain off, is it possible for me, or for anyone, to focus mental energy and bring the water to a boil through the power of the mind alone? If so, how would I go about developing this ability? 82.113.121.167 (talk) 11:15, 13 March 2010 (UTC)[reply]

No. If you can do it, and consistently, then get in touch with James Randi for your $1 million prize. --Mark PEA (talk) 11:28, 13 March 2010 (UTC)[reply]
You might consider asking this question on our yet to be created Wikipedia:Reference desk/Nonsense. DVdm (talk) 11:30, 13 March 2010 (UTC)[reply]
I believe this has been proposed. :) Zain Ebrahim (talk) 17:30, 13 March 2010 (UTC)[reply]
You must have lived such a sheltered live, DVdm. Here, for your edification, is an example of actual nonsense:
Or even:

(That's from the New Yorker).

By comparison, my question is extremely sensible, and very specific. You've lived a sheltered life. 82.113.121.167 (talk) 11:52, 13 March 2010 (UTC)[reply]
Well, I guess you can run a Stirling engine off the temperature difference of a brain and the environment and use that mechanical energy to drive a heat pump to concentrate heat to boil water, or convert it to electricity to drive an electric heater. It will require very many brains and probably the application of the Spherical cow theorem to work, though, and it's not what you would usually call "mental energy". It would, however, be a strong testament to the "power of the mind" ;-). --Stephan Schulz (talk) 11:36, 13 March 2010 (UTC)[reply]
See A. W. Underwood. He claimed to be able to set things on fire using the power of his mind (with only a _little_ assistance from white phosphorus). Paranormal powers do show a distressing tendency to have such explanations - I can't imagine why. ;) Tevildo (talk) 11:54, 13 March 2010 (UTC)[reply]
I can. - DVdm (talk) 12:00, 13 March 2010 (UTC)[reply]
Spontaneous human combustion Graeme Bartlett (talk) 12:29, 13 March 2010 (UTC)[reply]
:-) Quoting (emphasis mine):
  • "While there have been about 200 cited cases worldwide over a period of around 300 years, most of the alleged cases are characterised by the lack of a thorough investigation, or rely heavily on hearsay and oral testimony. In many of the more recent cases, where photographic evidence is available, it is alleged that there was an external source of heat present (often cigarettes), and nothing occurred "spontaneously.""
For the other desk. DVdm (talk) 13:30, 13 March 2010 (UTC)[reply]
What you don't understand, DVdm, is that in a scientific investigation, at least one time in a hundred, you are likely to see a statistically significant effect at the 99% confidence level. The only thing paranormal scientists now need to do, is take that 1 case out of 100, and somehow hone in on it, isolate it. That is the work that everyone who is rigorously interested in the paranormal must focus all of their mental energy on. 82.113.121.167 (talk) 14:20, 13 March 2010 (UTC)[reply]
Perhaps this is where I must emphasise that I'm not really all that "rigorously interested in the paranormal". I am however somewhat interested in the psychology of those who are. As this is a rather less interesting case, I think I'll sign off here. Sorry if was unable to help, but anyhow, the pleasure was mine :-) - DVdm (talk) 14:40, 13 March 2010 (UTC)[reply]
Presumably this would not be supernatural but indeed, some kind of natural phenomena. The brain really doesn't seem built to transmit the amount of energy required. It's also unclear to me how it would boil water at a distance without boiling the water in your head simultaneously (a microwave transmitter would not work if it was immersed in a bunch of water—it would heat itself). The only way I can imagine doing it (though I don't pretend to be the most imaginative) is to suppose that our brains are somehow tied to some very strange physics, the likes of which our pretty reliable theories have given no hints of, and of which an evolutionary explanation is conspicuously absent. One would also presumably expect that if concentrating on something could cause it to be physically manipulated or heated or whatever, that people who spend long periods of time in, say, solitary confinement, would have had ample time and opportunity to develop these abilities. Anyway—the answer seems pretty clearly to be "no, sorry," even if one does not necessarily believe that scientists "know everything." It simply seems implausible from a number of viewpoints, and there seems to be no reason to assume it is true other than "it would be cool" in a comic-book way.
Alas, don't be sad, your brain can accomplish much more than boiling water if you apply it in useful ways. Human brainpower, coupled with manpower and technology, have moved mountains, destroyed cities, healed millions. The human brain is a powerful thing in and of itself, with no need to appeal to the paranormal. --Mr.98 (talk) 15:28, 13 March 2010 (UTC)[reply]
This is exactly why I find it hard to believe. The human mind, of all the plants, mushrooms, and animals on Earth, literally in the millions, has the power to form coherent sentences, can formulate exquisitely abstract notions. The power of the human brain alone has allowed us to pierce the very fabric of the Universe and extract from it the secrets of electricity, which the power of the human brain has fully dominated and put at the command of every modern household; the power of the human brain has discovered and devised ways to apply atomic theory to split the atom, putting at our disposal not only vast amounts of energy in the form of nuclear fission power plants, far beyond what any plant or animal comes within many, many orders of magnitude of being able to produce, singly or in groups, but also what at this moment we are working on producing on Earth: the very process by which Sol our sun produces fusion energy, all invented, targeted, and worked upon through the power of the human brain; the human brain alone has cracked the secret of the atomic structure, the key to modern chemistry, and the human brain has realized ways of producing materials that have never appeared anywhere on Earth and have supremely useful functions for everyone. The human brain has devised magnetic levitation, wholly absent in all of nature, and uses it for locomotion at a speed that is literally orders of magnitude faster and at distances that are orders of magnitude farther than the fastest animal can sustain. Verily the human brain has cracked the cosmos; the genetic structure; problems of energy; of materials; of electricity and electronics; of land, air, sea, and space. Yet it cannot bring a pot of water to a boil? 82.113.121.167 (talk) 16:30, 13 March 2010 (UTC)[reply]
The mind can "bring a pot of water to boil", as it can do all those other things you listed. What it can't do is boil a pot of water alone, and as it happens, it can't do any of those other things alone either (one needs to be taught literacy skills to form sentences = mind not working alone). --Mark PEA (talk) 20:03, 13 March 2010 (UTC)[reply]
Again, sure it can. It's way of doing so is to instruct the body to take the pot to the stove and turn it on. --Stephan Schulz (talk) 16:52, 13 March 2010 (UTC)[reply]
Yes, we human are very clever. However, there's no reason to think that the ability to do complex maths or think in the abstract would also give the power of telekinesis. In a nutshell, just because you can imagine it (or you really, really want it), does not make it so. – ClockworkSoul 16:53, 13 March 2010 (UTC)[reply]
Right, we can boil water just fine, in a dozen different ways. The mind does this. It understands heat and fire and boiling points and everything. But it cannot impose its intentions on the world with sheer thought alone. In every case of it "doing" something, it uses a medium to translate computation into physical effect—be in hands, voice, movement of feet. It is not set up to broadcast radio waves or microwaves or lasers or large amounts of heat. Neither is your computer. The human brain computes. It does a great job of that. As I.I. Rabi said in a different context: "What more do you want, mermaids?" --Mr.98 (talk) 19:33, 13 March 2010 (UTC)[reply]

Studies show that the human brain consumes oxygen from the bloostream at a rate that is equivalent to 100 Watts. However, the activity actually conducted by the brain accounts for only about 10 Watts. The remaining energy is lost as electrical signals. These can be measured by electroencephalography (the EEG machine everyone has seen). Different wave patterns are generated by different types of brain activity. Some individuals have trained themselves to focus this brain activity in such a way that the wave pattern matches the vibrational frequency of the hydrogen-oxygen sigma bonds in water. This matching resonance frequency increases the rate of vibration, and of course this type of intramolecular vibration manifests itself as heat. The 90 Watts of power lost from the brain as electrical waves and focused on H-O bonds is more than enough to boil a liter of water. 72.94.164.21 (talk) 16:58, 13 March 2010 (UTC) (Since DVdm's WP:RD/N doesn't exist yet, I decided to post here instead.)[reply]

The whole human body only uses 100 watts of power. The brain consumes 20-40 watts, but there's no way to "account" for any of it; our brains can be as inefficient as they want to be for the computations they perform. You could warm water with a 30 watt lightbulb just fine from the waste heat. But brain waves are super-weak; that's why you have to attach electrodes to someone's head just to detect them.
If brain waves were the right frequency, and strong enough, to boil water, we'd be in trouble, because the brain is mostly water... It would be a neat ability, though, other than that. Paul Stansifer 17:21, 13 March 2010 (UTC)[reply]
Right. I don't see how the brain is going to boil water without boiling itself. A microwave would not work if it was immersed in water, presumably (in the sense that it would not be effective exclusively for a kettle of water kept at a distance). --Mr.98 (talk) 19:33, 13 March 2010 (UTC)[reply]
Side discussion moved to talk pageClockworkSoul 18:08, 13 March 2010 (UTC)[reply]
...specifically, to here: Wikipedia_talk:Reference_desk#Nonsense_questions. StuRat (talk) 19:52, 13 March 2010 (UTC)[reply]
How about if we lower the pressure to where water boils right around body temp, put the person's skull in contact with the water, then have them concentrate really hard on some difficult problem ? Couldn't we then get their head to give off ever so slightly more heat, and therefore cause the water to boil ? (Of course, we'd probably need to provide a higher than normal percentage of oxygen in the air, to compensate for the lower pressure.) StuRat (talk) 20:06, 13 March 2010 (UTC)[reply]
The pressure would have to be about 6% of the atmospheric pressure. I think I would have a hard time thinking under those circumstances. Why can't we give the OP the honest answer and let him know that the whole thing is ludicrous? Dauto (talk) 21:19, 13 March 2010 (UTC)[reply]
Wikipedia has an article on a relevant topic so far unmentioned: Pyrokinesis which is a kind of Psychokinesis. Graeme Bartlett (talk) 22:48, 13 March 2010 (UTC)[reply]
I doubt there's any direct credible scientific proof on this, but brain waves may be shown to have effects external to the brain itself. For example a robot that places balls into a container within a random number of moves may be "affected" by human thought causing it to reduce its number of moves (but correlation with brain activity is not shown directly). ~AH1(TCU) 23:18, 13 March 2010 (UTC)[reply]
Um, what? Citation needed. --Mr.98 (talk) 00:08, 14 March 2010 (UTC)[reply]
I'm sorry, what did that comment add to this discussion? You say that something might happen but you doubt there is any evidence for it. You don't even claim that anyone before you has claimed it does happen or that you have observed it to happen. You are just knowingly spouting nonsense for the sake of it... --Tango (talk) 05:13, 14 March 2010 (UTC)[reply]

Radio waves

Resolved

Is there a difference between radio signals and radio waves? —Preceding unsigned comment added by Fire2010 (talkcontribs) 20:26, 13 March 2010 (UTC)[reply]

The signal is the message that is being transmitted via waves. Zain Ebrahim (talk) 20:39, 13 March 2010 (UTC)[reply]
I not sure I understand. Can "signal" and "wave" both be used when describing the transmissions and receiving of music or television channels? Are they essentially the same concept with different names or are they totally different from each other? —Preceding unsigned comment added by Fire2010 (talkcontribs) 20:45, 13 March 2010 (UTC)[reply]
Yes, both words can be used when talking about TV/radio. So the signal (which is the message) could be a TV show and your television receives the signal sent from the TV company in the form of waves. So the signal is transmitted as waves. Does that make sense? Zain Ebrahim (talk) 21:03, 13 March 2010 (UTC)[reply]
The signal is usually encoded either as modulations of the radio wave amplitude (AM broadcasting) or as modulations of the radio wave frequency (FM broadcasting). Dauto (talk) 21:13, 13 March 2010 (UTC)[reply]
To help understand the difference, consider the SETI astronomers, who scan the skies looking at radio waves generated by astronomical objects, hoping to find a radio signal (from an alien intelligence) among them. StuRat (talk) 21:11, 13 March 2010 (UTC)[reply]
I think (feel free to correct me if I'm wrong) that the difference would be that a radio signal would be a radio wave that carries information. Vimescarrot (talk) 21:51, 13 March 2010 (UTC)[reply]


Thanks everyone, I think I understand now. The "signal" is the message, and the "wave" is how it arrives. Thanks! —Preceding unsigned comment added by Fire2010 (talkcontribs)

acid reflux

This question has been removed. Per the reference desk guidelines, the reference desk is not an appropriate place to request medical, legal or other professional advice, including any kind of medical diagnosis, prognosis, or treatment recommendations. For such advice, please see a qualified professional. If you don't believe this is such a request, please explain what you meant to ask, either here or on the Reference Desk's talk page.
This question has been removed. Per the reference desk guidelines, the reference desk is not an appropriate place to request medical, legal or other professional advice, including any kind of medical diagnosis or prognosis, or treatment recommendations. For such advice, please see a qualified professional. If you don't believe this is such a request, please explain what you meant to ask, either here or on the Reference Desk's talk page. --~~~~
See WT:RD#Medremoval. Zain Ebrahim (talk) 20:44, 13 March 2010 (UTC)[reply]

Feigning death drug

I just watched an episode of the TV series "The New Avengers" - a guy injects a substance to feign death, later wakes and says "a drug that suspends the heart and the respiration" - isn't that impossible (brain death?) or do such substances exist? Thanks for info., --AlexSuricata (talk) 23:24, 13 March 2010 (UTC)[reply]

Well, potassium chloride stops the heart and thus causes death by cardiac arrest, yes. I believe the mythical drug you're describing sort of freezes the body's operations, allowing them to resume hours later, and the subject only has a hangover, presumably. The same mythical drug was in The Dark Knight Returns graphic novel. No, no such drug exists; as you note, a stoppage of the heart and breathing does kill people. On a related note, the field of cryonics seeks to put people or other animals in suspended animation, but by means of chilling them. Comet Tuttle (talk) 00:37, 14 March 2010 (UTC)[reply]
(edit conflict) Yes, it's impossible. You would suffer massive brain damage due to lack of oxygen to the brain. That could be reduced by inducing hypothermia, but even that would be very unreliable. There are ways to significantly reduce the heart-rate and respiration, but not eliminate them entirely. Someone would appear dead at a glance, but not if someone actually made a significant effort to check (hold a mirror in front of their mouth for a minute, say). --Tango (talk) 00:38, 14 March 2010 (UTC)[reply]
The toxin in the blue-ringed octopus paralyzes and can make a person appear dead, especially when they stop breathing. However, if artificial respiration is used until the toxin clears their system, they can recover. So, while the heart continues to beat, the lack of any breathing could indeed make others assume the person is dead. StuRat (talk) 03:16, 14 March 2010 (UTC)[reply]

March 14

Is it possible to cause a prolonged temperature increase (eben a small one) in the skull just through mental activity?

Is it possible to increase the temperature of the skull, even by only a slight amount, just through assiduous mental activity? If so, what kind of mental activity is appropriate that I could do to show this effect? (Note: I'm the poster who asked about boiling water above) 80.187.97.42 (talk) 00:32, 14 March 2010 (UTC)[reply]

A metabolic PET scan can show increased glucose oxidation occurring in parts of the brain where mental activity is occurring, but actual temperature increases are too small and thermal conductivity of human tissue too poor to measure any differences overlying the scalp. (And yes, it was obvious it was the same questioner.) alteripse (talk) 05:14, 14 March 2010 (UTC)[reply]

A silly question

I've noticed whenever I deliver rolled up papers that they are either rolled up clockwise or counterclockwise, depending upon the end of the paper I am looking at. On the other hand whenever I read a astronomy article in order to improve my brain I only seem to get more confused since the articles say that Galactic spirals only rotate in a clockwise direction. However, when you view the Galaxy from the other side, like looking at the other end of the rolled up papers, the Galaxy is spiraling counterclockwise instead of clockwise. Should I stop reading astronomy articles and just go with what I know? 71.100.11.118 (talk) 02:07, 14 March 2010 (UTC)[reply]

A spiral galaxy will look clockwise from one direction and anti-clockwise from the other, just like any other spiral. Can you give an example of one of the astronomy articles you have been reading that say otherwise? Perhaps you are misunderstanding them (either that, or you need to read better astronomy articles - we can help you there, too!). --Tango (talk) 03:25, 14 March 2010 (UTC)[reply]
The latest article (2002) I've read makes a comparison of the rotation of the central bulge and the rotation of the spiral arms as being in opposite directions. What is surprising and confusing to me is that I can find no characteristic of classification which is based on the 3-axis orientation of the Galaxies and in conjunction on clockwise or counterclockwise rotation. 71.100.11.118 (talk) 04:09, 14 March 2010 (UTC)[reply]
I can't comment on the galaxy described in that article, having not researched it, but that article doesn't say anything about galaxies being rolled up the same way from both sides... --Tango (talk) 05:09, 14 March 2010 (UTC)[reply]
Clockwise and anti-clockwise rotation as seen from Earth has been counted. See Galaxy Zoo#Progress. PrimeHunter (talk) 03:49, 14 March 2010 (UTC)[reply]

Brain acceleration?

I was staring at clock, and noticed that when I first look at it, it seems a second hand stays there for a while (more than a second), and only then it goes on at a normal pace. Same thing happens with digital clocks as well. Why does it happen? Does the brain accelerate itself in order to analyze the information given by clock, so it seems time slows down for a while? 195.238.106.80 (talk) 02:19, 14 March 2010 (UTC)[reply]

Interesting. I have never noticed this myself, but a bit of Googling shows that it is a known phenomenon, although the cause is not clearly understood. Here is a review from last year that discusses the effect, along with other temporal illusions. Looie496 (talk) 03:08, 14 March 2010 (UTC)[reply]
Just to add a bit of info, the review suggests that this is a consequence of the suppression of visual perception that occurs during saccadic eye movements -- when you move your eyes rapidly from one place to another, your visual system basically switches off during the movement, but your time perception is warped for a short time before and afterward so that you don't perceive any gap -- in effect you perceive the eye movement to occur instantaneously. Looie496 (talk) 03:14, 14 March 2010 (UTC)[reply]
It's an amazing thing that our brains can create a seemingly seamless perception of the world given that the information provided by the senses have so many gaps and delays and mismatches. Dauto (talk) 03:59, 14 March 2010 (UTC)[reply]
There is a really interesting article in this month's PopSci about this kind of stuff, entitled "How Time Flies". It is related to the brain's perception of time dilation under different circumstances and, while nothing is conclusive at this point, it elaborates on some pretty plausible theories. However, since it is the current issue, they won't have it posted on their web site for another couple weeks. —Akrabbimtalk 04:08, 14 March 2010 (UTC)[reply]
This article in New Scientist is good, and discusses the effect extensively: http://www.newscientist.com/article/mg20427311.300-timewarp-how-your-brain-creates-the-fourth-dimension.html --99.237.234.104 (talk) 04:57, 14 March 2010 (UTC)[reply]

Temperature of a high-gravity planet?

Say that there is a planet with a very strong gravitational pull. Would the gravity condense some liquids into solid form, meaning there is more ice on the surface? Or would it cause more heat due to friction? If the strength of the planet's gravitational pull was the only differing factor, would the resulting surface conditions be more likely to be hotter, colder, or about the same as an Earth-like planet? 97.104.210.67 (talk) 05:57, 14 March 2010 (UTC)[reply]

Gravity won't condense anything, pressure does that. Gravity is a factor in determining pressure, but it isn't the only one. Venus' surface pressure is about 90 times that of Earth despite having approximately the same surface gravity. --Tango (talk) 06:06, 14 March 2010 (UTC)[reply]
Excuse me for my mistake, I should have remembered that, but it is 1 AM my time. I'm just curious to know which result is more likely to occur. 97.104.210.67 (talk) 06:09, 14 March 2010 (UTC)[reply]